Vous êtes sur la page 1sur 73

Livre du professeur - Mathématiques Tle Spécialité - Chapitre 4 : Suites

Livre du professeur - Mathématiques


Chapitre 4 : Suites

Table des matières


1 Informations sur ce chapitre 2

2 Avant de commencer 3
2.1 Corrigés des exercices . . . . . . . . . . . . . . . . . . . . . . . . . . . . . . 3

3 Activités 6
3.1 Activité A : Ne pas faire les choses à moitié . . . . . . . . . . . . . . . . . 6
3.2 Activité B : Ne pas dépasser les limites . . . . . . . . . . . . . . . . . . . . 8
3.3 Activité C : Évolution d’une population de bactéries . . . . . . . . . . . . . 8
3.4 Activité D : Une suite bien encadrée . . . . . . . . . . . . . . . . . . . . . 9

4 Auto-évaluation 11

5 TP/TICE 14
5.1 Corrigé du TP 1 : Étude de la convergence des séries de Riemann . . . . . 14
5.2 Corrigé du TP 2 : Approximation de π par la méthode d’Archimède . . . . 17

6 Travailler les automatismes 21


6.1 Exercices à l’oral . . . . . . . . . . . . . . . . . . . . . . . . . . . . . . . . 21
6.2 Exercices . . . . . . . . . . . . . . . . . . . . . . . . . . . . . . . . . . . . . 21

7 Exercices d’entraînement partie 1 32

8 Exercices d’entraînement partie 2 38

9 Exercices d’entraînement partie 3 42

10 Exercices d’entraînement partie 4 50

11 Exercices de synthèse 55

12 Exercices Préparer le bac 69

Document sous licence libre Creative Commons


2

Livre du professeur - Mathématiques Tle Spécialité - Chapitre 4 : Suites

1 Informations sur ce chapitre


Les suites ont été introduites en classe de Première et sont un outil indispensable pour
modéliser des phénomènes discrets avec des utilisations dans plusieurs autres disciplines.
Dans ce chapitre on formalise l’étude du comportement asymptotique des suites amorcé
en classe de Première. Pour cela, on commence avec l’étude des suites convergentes et
la notion de limite finie. Dans un premier temps, nous reviendrons systématiquement
à la définition pour prouver qu’une suite converge. Ensuite, nous pourrons recourir au
théorème de convergence monotone en faisant bien remarquer aux élèves que, bien qu’il
permette de prouver la convergence d’une suite, il ne permet pas en général de déterminer
sa limite.
Nous passerons dans un second temps à l’étude des suites ayant une limite infinie. De façon
analogue, nous prouverons qu’une suite diverge vers ±∞ en se ramenant à la définition.
Nous pourrons ensuite traiter du cas particulier des suites monotones.
Dans un troisième temps, nous allons mettre en place les théorèmes d’opérations sur les
limites de façon progressive, en commençant par la limite d’une somme de deux suites,
puis la limite d’un produit de deux suites et enfin la limite d’un quotient de deux suites.
Il est important de faire comprendre aux élèves que certaines limites ne peuvent pas être
calculées en utilisant directement ces théorèmes et on montre que l’on ne peut pas conclure
dans le cas général. Lorsque ces cas se rencontrent, on parlera de façon traditionnelle de
« Forme Indéterminées ». Viendra ensuite un temps où les élèves vont devoir acquérir
certains automatismes afin de lever l’indétermination, notamment en factorisant par le
terme de plus haut degré.
Dans un dernier temps, nous étudierons la convergence de suites à l’aide des théorèmes de
comparaison (pour montrer qu’une suite diverge vers ±∞) ou du théorème des gendarmes
(pour montrer qu’une suite converge).
Ce chapitre est aussi l’occasion de travailler les différents types de raisonnements, notam-
ment le raisonnement par récurrence et la recherche de contre-exemples.

Document sous licence libre Creative Commons


3

Livre du professeur - Mathématiques Tle Spécialité - Chapitre 4 : Suites

2 Avant de commencer
2.1 Corrigés des exercices
Corrigé exercice 1 :
1. u1 = 3 × 1 + 5 = 8
u2 = 3 × 2 + 5 = 11
u3 = 3 × 3 + 5 = 14
u5 = 3 × 5 + 5 = 20

2. u1 = 21 − 1 = 1
u2 = 22 − 1 = 3
u3 = 23 − 1 = 7
u5 = 25 − 1 = 31
1 1
3. u1 = =
1+1 2
2 2
u2 = =
2+1 3
3 3
u3 = =
3+1 4
5 5
u5 = =
6 6
4. Pour les suites définies par récurrence, on constate que le calcul de u4 est nécessaire
pour calculer u5 même si ce n’est pas explicitement demandé. u1 = 2 × u0 + 7 =
2×1+7=9
u2 = 2 × u1 + 7 = 2 × 9 + 7 = 25
u3 = 2 × u2 + 7 = 2 × 25 + 7 = 57
u4 = 2 × u3 + 7 = 2 × 57 + 7 = 121 u5 = 2 × u4 + 7 = 2 × 121 + 7 = 249

5. u1 = 3 × u0 − 2 × 0 = 3 × 2 − 2 × 0 = 6
u2 = 3 × u1 − 2 × 1 = 3 × 6 − 2 × 1 = 16
u3 = 3 × u2 − 2 × 2 = 3 × 16 − 2 × 2 = 44
u4 = 3 × u3 − 2 × 3 = 3 × 44 − 2 × 3 = 126
u5 = 3 × u4 − 2 × 4 = 3 × 126 − 2 × 4 = 370

Corrigé exercice 2 :
1. tn+1 = 2(n + 1) + 4 = 2n + 2 + 4 = 2n + 6
tn−1 = 2(n − 1) + 4 = 2n − 2 + 4 = 2n + 2
t2n = 2(2n) + 4 = 4n + 4
t3n−2 = 2(3n − 2) + 4 = 6n − 4 + 4 = 6n

Document sous licence libre Creative Commons


4

Livre du professeur - Mathématiques Tle Spécialité - Chapitre 4 : Suites

2. tn+1 = (n + 1)2 − (n + 1) + 1 = n2 + 2n + 1 − n − 1 + 1 = n2 + n + 1
tn−1 = (n − 1)2 − (n − 1) + 1 = n2 − 2n + 1 − n + 1 + 1 = n2 − 3n + 3
t2n = (2n)2 − 2n + 1 = 4n2 − 2n + 1
t3n−2 = (3n − 2)2 − (3n − 2) + 1 = 9n2 − 12n + 4 − 3n + 2 + 1 = 9n2 − 15n + 7
(n + 1) − 2 n−1
3. tn+1 = =
(n + 1) + 1 n+2
(n − 1) − 2 n−3
tn−1 = =
(n − 1) + 1 n
2n − 2
t2n =
2n + 1
(3n − 2) − 2 3n − 4
t3n−2 = =
(3n − 2) + 1 3n − 1

Corrigé exercice 3 :
1. La fonction f : x 7→ 4x − 5 est une fonction affine dont le coefficient directeur est
strictement positif. Elle est donc croissante sur [0; +∞[. Pour tout n ∈ N on a
vn = f (n), donc la suite (vn ) est croissante.
 
2 2 2 2
2. Pour tout n > 1, on a vn+1 − vn = 1 + − 1+ = 1+ −1−
n+1 n n+1 n
2 2 2n − 2n − 2 2
= − = = − . Or, pour tout n > 1, on a n > 0 et
n+1 n n(n + 1) n(n + 1)
n + 1 > 0 d’où n(n + 1) > 0.
Donc vn+1 − vn 6 0 et donc la suite (vn ) est décroissante.
2n+1
vn+1 2n+1 n 2n
3. Pour tout n > 1, on a vn > 0 et = n +n 1 = × n = .
vn 2 n+1 2 n+1
n
2n
Or, pour tout n > 1, on a 2n > n + 1 d’où > 1.
n+1
vn+1
Donc > 1 et donc la suite (vn ) est croissante.
vn
4. La fonction f : x 7→ (x − 5)2 est une fonction polynôme du second degré donnée sous
forme canonique. Elle est décroissante sur [0; 5] et croissante sur [5; +∞[.
Comme, pour tout n ∈ N, on a vn = f (n), alors la suite (vn ) est croissante à partir
de n = 5.

Corrigé exercice 4 :
1. Pour tout n ∈ N, on a wn = w0 + n × r = 2 − 3n.

2. Pour tout n ∈ N, on a wn = w0 + n × r = 18 + 5n.


3 1 1 1
3. Pour tout n ∈ N∗ , on a wn = w1 + (n − 1) × r = + (n − 1) = + n.
4 2 4 2
Document sous licence libre Creative Commons
5

Livre du professeur - Mathématiques Tle Spécialité - Chapitre 4 : Suites

Corrigé exercice 5 :
1. Pour tout n ∈ N, on a pn = p0 × q n = 3 × 4n .
1
2. Pour tout n ∈ N, on a pn = p0 × q n = × (−2)n = −(−2)n−1 .
2
 n−1
∗ n−1 1
3. Pour tout n ∈ N , on a pn = p1 × q =5× .
2

Corrigé exercice 6 :
1. u0 = −2
u1 = 0, 5 × u0 + 3 = 0, 5 × (−2) + 3 = −1 + 3 = 2
u2 = 0, 5 × u1 + 3 = 0, 5 × 2 + 3 = 1 + 3 = 4

2. a. Pour tout entier naturel n, on a vn+1 = un+1 − 6 = 0, 5un + 3 − 6 = 0, 5un − 3 =


0, 5(un − 6) = 0, 5vn . Donc (vn ) est une suite géométrique de raison q = 0, 5 et
de premier terme v0 = u0 − 6 = −2 − 6 = −8.
b. Ainsi, pour tout entier naturel n, on a vn = v0 × q n = −8 × 0, 5n .
c. Donc, pour tout entier naturel n, on a un = vn + 6 = −8 × 0, 5n + 6.

Corrigé exercice 7 :
1. En 2020, il y a u1 = 250 × 0, 9 + 35 = 260 élèves inscrits dans l’école de musique.
En 2021, il y a u2 = 260 × 0, 9 + 35 = 269 élèves inscrits dans l’école de musique.

2. Comme u1 − u0 = 260 − 250 = 10 et u2 − u1 = 269 − 260 = 9, on en déduit que la


u1 260 u2 269
suite (un ) n’est pas arithmétique. Comme = = 1, 04 et = ≈ 1, 03,
u0 250 u1 260
on en déduit que la suite (un ) n’est pas géométrique.

3. Pour tout n ∈ N, on a un+1 = 0, 9 × un + 35 et u0 = 250. D’après le tableau de


valeur de la calculatrice, la suite (un ) semble être croissante.

4. 2050 correspond à l’année 2019 + 31. Grâce à la calculatrice, on trouve u31 ≈ 346.

Document sous licence libre Creative Commons


6

Livre du professeur - Mathématiques Tle Spécialité - Chapitre 4 : Suites

3 Activités
3.1 Activité A : Ne pas faire les choses à moitié
Corrigé activité A :
Questions :
1. L’aire totale du rectangle est 1 × 2 = 2 m2. A1 est l’aire totale coloriée après une
2
seule étape. Ainsi, on a A1 = = 1.
2
1
L’aire non colorée est donc de 1 m2. Ainsi, A2 = A1 + = 1, 5.
2
0, 5
L’aire non colorée est alors de 0,5 m2. Ainsi, A3 = A2 + = 1, 75.
2
2. Lors de l’étape n, il reste à colorier une surface égale à 2 − An . Lors de l’étape
n + 1, on colorie la moitié de cette surface restante et l’aire totale coloriée est donc
1 1 1 1
An+1 = An + (2 − An ) = An + × 2 − An = 1 + An .
2 2 2 2
3. a. On remarque que les termes de la suite se rapprochent de la valeur 2.

b. Si on continue d’afficher les termes de la suite (An ) bien au-delà de n = 50,


on constate qu’ils se rapprochent toujours de 2 (la calculatrice indique même
rapidement qu’ils sont égaux à 2). Cela n’est pas étonnant puisqu’avec le pro-
cédé de coloriage, on colorie une surface qui se rapproche de plus en plus de la
surface totale du rectangle.
4. On définit une nouvelle suite dans la calculatrice.

Document sous licence libre Creative Commons


7

Livre du professeur - Mathématiques Tle Spécialité - Chapitre 4 : Suites

C’est à partir du rang 5 que l’on a |A − An | < 0, 1.

C’est à partir du rang 8 que l’on a |A − An | < 0, 01.

Au bout de la cinquième étape, l’aire restante est inférieure à 0, 1 m2. Au bout de


la huitième étape, l’aire restante est inférieure à 0, 01 m2.

5. Avec ce procédé de coloriage, il reste toujours une surface non-coloriée, quelque soit
l’étape à laquelle on se place. Ainsi l’aire de la surface coloriée ne sera jamais égale
à A. Cependant, la surface restante devient aussi proche de 0 que l’on veut (sans
jamais atteindre 0).

Bilan : La suite (An ) possède une limite finie car les termes de la suite semble devenir
aussi proches de 2 que l’on veut.

Document sous licence libre Creative Commons


8

Livre du professeur - Mathématiques Tle Spécialité - Chapitre 4 : Suites

3.2 Activité B : Ne pas dépasser les limites


Corrigé activité B :
Questions :
p1 p2 p1  p1  p2
1. On a a1 = A × et a2 = a1 + (A − a1 ) × = A× + A−A× ×
100 100 100 100 100
100(p1 + p2 ) − p1 p2
=A× .
10 000
2. Il nous manque les valeurs de p1 et p2 pour calculer a1 et a2 .

3. La suite (an ) est croissante car, à chaque étape, on ajoute une nouvelle surface
colorée à la surface déjà colorée.

4. L’aire totale colorée ne peut pas être supérieure à A donc nécessairement, an 6 A.

5. On peut supposer que, lorsque n tend vers l’infini, la suite (an ) va tendre vers A.

Bilan : On sait que la surface totale colorée ne peut pas dépasser A et on peut supposer
qu’au bout d’un grand nombre d’étapes, on aura fini par colorier toute l’étoile. Cela justifie
que la suite (an ) admet une limite finie.

3.3 Activité C : Évolution d’une population de bactéries


Corrigé activité C :
Questions :

1. Augmenter de 3 % revient à multiplier par 1,03. b1 = b0 ×1, 03 = 1000×1, 03 = 1030


b2 = b1 × 1, 03 = 1030 × 1, 03 ≈ 1061

2. Pour tout entier naturel n, on a bn+1 = bn × 1, 03.

3. (bn ) est donc une suite géométrique de raison q = 1, 03 et de premier terme b0 =


1000.

4. a. On calcule les termes de la suite (bn ) grâce à la calculatrice.

On obtient qu’au bout de 24 jours la population de bactéries aura doublé.

Document sous licence libre Creative Commons


9

Livre du professeur - Mathématiques Tle Spécialité - Chapitre 4 : Suites

b. De même, on obtient qu’au bout de 38 jours la population de bactéries aura


dépassé les 3 000 individus.

5. a. N ← 0
B ← 1000
Tant que B 6 S :
N ←N +1
B ← B × 1, 03
Fin Tant que
Afficher N
b. Pour déterminer au bout de combien de jours il y aura plus de 5 000 bactéries,
on remplace S par 5 000 dans l’algorithme.
D’après la calculatrice, c’est au bout de 55 jours que la population de bactéries
aura dépassé les 5 000 individus (on pourrait aussi programmer cet algorithme
avec Python pour obtenir le résultat).

6. a. Le nombre d’individus de cette population semble être strictement croissant.


b. Quelle que soit la valeur de M choisie, il semble qu’en attendant assez long-
temps, le nombre de bactéries le dépasse.

Bilan : Quel que soit le nombre M choisi, le nombre de bactéries de cette population
semble pouvoir le dépasser si on attend assez longtemps. Ainsi, on peut dire que la suite
(bn ) a pour limite +∞ lorsque n tend vers +∞. Cette suite ne peut pas être bornée.

3.4 Activité D : Une suite bien encadrée


Corrigé activité D :
Questions :
1
1. La fonction x 7→ est décroissante sur ]0; +∞[ donc la suite (un ) est décroissante.
x
De plus, on a lim un = 0.
n→+∞
−1
De même, la fonction x → 7 est croissante sur ]0; +∞[ donc la suite (vn ) est
x
croissante. De plus, on a lim vn = 0.
n→+∞

Document sous licence libre Creative Commons


10

Livre du professeur - Mathématiques Tle Spécialité - Chapitre 4 : Suites

2. À l’aide de la calculatrice, on constate que la suite (wn ) n’est pas monotone.


−1 cos(n) 1
3. Pour tout entier naturel n > 1, on a −1 6 cos(n) 6 1. D’où 6 6 ,
n n n
car n est strictement positif. Par conséquent, pour tout entier naturel n > 1, vn 6
w n 6 un .

4. On obtient le résultat ci-dessous. Les points verts représentent la suite (wn ), les
points bleus la suite (un ) et les points rouges (qui sont bien cachés), la suite (vn ).

5. La limite de la suite (wn ) semble être 0.

6. Pour tout entier naturel n > 1, on a −1 6 sin(n) 6 1 d’où −3 6 3 sin(n) 6 3.


−3 3 sin(n) 3 −3 3 sin(n) 3
On a alors 6 6 et +26 + 3 6 + 2.
n n n n n n
−3 3
Ainsi en posant, pour tout entier naturel n > 1, yn = + 2 et xn = + 2, on a
n n
yn 6 zn 6 xn . Or, la suite (xn ) est décroissante et lim xn = 2 par opérations sur
n→+∞
les limites. De même, la suite (yn ) est croissante et lim yn = 2 par opérations sur
n→+∞
les limites. Ainsi, on peut supposer que lim zn = 2.
n→+∞

Bilan : On encadre la suite dont on cherche à déterminer la limite par deux autres
suites ayant toutes deux la même limite. On conjecture ensuite que la limite de la suite
encadrée est la même que celle des deux autres suites.

Document sous licence libre Creative Commons


11

Livre du professeur - Mathématiques Tle Spécialité - Chapitre 4 : Suites

4 Auto-évaluation
Corrigé exercice 8 :
 
2 2
n 3+ 3 +
3n + 2 n n . Or, lim 2 = 0
Pour tout entier naturel n, on a un = =   =
5 + 2n 5 5 n→+∞ n
n +2 +2
n n
2 5 5
d’où, par somme, lim 3+ = 3. De même, lim = 0 d’où, par somme, lim +2 =
n→+∞ n n→+∞ n n→+∞ n
3
2. Ainsi, par quotient, lim un = .
n→+∞ 2
Réponse : b.

Corrigé exercice 9 :
En représentant les premiers termes de la suite (vn ) sur la calculatrice, on remarque qu’elle
n’est ni croissante, ni décroissante, ni convergente. Elle semble bornée entre −10 et 30.

Réponse : a.

Corrigé exercice 10 :
1
La suite définie pour tout entier n > 1 par un = est majorée par 4 et converge vers un
n
réel `. Pourtant, aucune information nous permet d’affirmer que ` = 4, ni que (un ) est
croissante ou encore que (un ) constante égale à 4 à partir d’un certain rang.
Réponse : c.

Corrigé exercice 11 :
• Si (un ) est la suite définie pour tout entier naturel n par un = n2 et (vn ) est la suite
définie pour tout entier naturel n par vn = −n, on a lim un = +∞, lim vn = −∞
n→+∞ n→+∞
un un n2
et lim = −∞ car, pour tout entier n > 1, on a = = −n.
n→+∞ vn vn −n
• Si (un ) est la suite définie pour tout entier naturel n par un = n et (vn ) est la suite
définie pour tout entier naturel n par vn = −n, on a lim un = +∞, lim vn = −∞
n→+∞ n→+∞
un un n
et lim = −1 car, pour tout entier n > 1, on a = = −1.
n→+∞ vn vn −n
Document sous licence libre Creative Commons
12

Livre du professeur - Mathématiques Tle Spécialité - Chapitre 4 : Suites

On ne peut donc pas conclure dans le cas général.


Réponse : d.

Corrigé exercice 12 :
À partir d’un certain rang, tous les termes d’une suite qui converge vers 1 sont contenus
dans un intervalle ouvert contenant 1. Ainsi, la suite est bien bornée à partir d’un certain
rang.
En particulier, en prenant comme intervalle ]0; 1 + ε[ où ε est un réel positif, tous les
termes de la suite sont positifs à partir d’un certain rang.
En revanche, cette suite n’est ni croissante ni majorée par 1.
(−1)n
Par exemple, la suite (un ) définie pour tout entier n > 1 par un = + 1 converge
n
vers 1 mais elle n’est ni croissante, ni majorée par 1.
Réponses : b. et c.

Corrigé exercice 13 :
Considérons la suite (vn ) définie pour tout entier naturel n par vn = (−1)n . Elle est bornée
entre −1 et 1 mais n’est ni monotone, ni convergente.
1
En revanche, la suite (vn ) définie pour tout entier n > 1 par vn = est bornée entre 0 et
n
1. De plus, elle est décroissante et convergente.
On ne peut donc pas conclure dans le cas général.
Réponse : d.

Corrigé exercice 14 :
Considérons les suites (un ) et (vn ) définies pour tout entier n > 1 par :
un
• un = n2 et vn = n. On a lim = +∞.
n→+∞ vn

un
• un = n et vn = n2 . On a lim = 0.
n→+∞ vn

un
• un = n et vn = n. On a lim = 1.
n→+∞ vn

un
En revanche, on ne peut jamais avoir lim = −∞.
n→+∞ vn
Réponses : a., b. et c.

Corrigé exercice 15 :
a. C’est le théorème de comparaison.

b. C’est encore le théorème de comparaison.

c. Considérons les suites (un ) et (vn ) définies pour tout entier naturel n par un = 1 et
vn = n. On a bien un 6 vn pour tout entier naturel n et lim vn = +∞. Pourtant,
n→+∞
lim un = 1.
n→+∞

Document sous licence libre Creative Commons


13

Livre du professeur - Mathématiques Tle Spécialité - Chapitre 4 : Suites

d. Considérons les suites (un ), (vn ) et (wn ) définies pour tout entier n > 1 par un =
1 1
−1 − , vn = 1 et wn = 2 + . On a bien un 6 vn 6 wn pour tout entier n > 1 et
n n
lim un < 0 et lim wn > 0. Pourtant, lim vn = 1.
n→+∞ n→+∞ n→+∞

Réponses : a. et b.

Corrigé du problème 16 :
1. a. Pour tout entier naturel n, on a : vn+1 = un+1 − 550
= 0, 96un + 22 − 550
= 0, 96un − 528
 
528
= 0, 96 un −
0, 96
= 0, 96 (un − 550)
= 0, 96vn
Donc la suite (vn ) est géométrique de raison q = 0, 96 et de premier terme
v0 = u0 − 550 = −250.
b. Pour tout entier naturel n, on a vn = v0 × q n = −250 × 0, 96n .
c. D’où un = 550 + vn = 550 − 250 × 0, 96n .

2. Comme −1 < 0, 96 < 1 alors lim 0, 96n = 0. D’où, par produit, lim −250 ×
n→+∞ n→+∞
0, 96n = 0. Ainsi, par somme, lim un = lim 550 − 250 × 0, 96n = 550.
n→+∞ n→+∞

Document sous licence libre Creative Commons


14

Livre du professeur - Mathématiques Tle Spécialité - Chapitre 4 : Suites

5 TP/TICE
5.1 Corrigé du TP 1 : Étude de la convergence des séries de
Riemann
Méthode 1
1. Le programme complété est le suivant.

2. a. Pour α = 1, la suite (Sn ) semble diverger vers +∞.


b. Pour α = 1, 1, la suite (Sn ) semble converger.
c. Pour α = 0, 5, la suite (Sn ) semble diverger vers +∞.
d. Pour α = 2, la suite (Sn ) semble converger.
e. Pour α = 0, 8, la suite (Sn ) semble diverger vers +∞.
f. Pour α = 20, la suite (Sn ) semble converger.
g. Pour α = 10, la suite (Sn ) semble converger.
h. Pour α = 0, 1, la suite (Sn ) semble diverger vers +∞.

3. Il semble que la suite (Sn ) converge lorsque α > 1 et diverge vers +∞ lorsque α 6 1.

Méthode 2
1. Il suffit d’entrer la valeur 4 dans la cellule A1.

2. On rentre les valeurs dans la colonne C.

Document sous licence libre Creative Commons


15

Livre du professeur - Mathématiques Tle Spécialité - Chapitre 4 : Suites

3. a. On a S1 = 1.
b. Voici la formule à entrer.

4. On obtient le résultat suivant :

5. a. Pour α = 1, la suite (Sn ) semble diverger vers +∞.


b. Pour α = 1, 1, la suite (Sn ) semble converger.
c. Pour α = 0, 5, la suite (Sn ) semble diverger vers +∞.
d. Pour α = 2, la suite (Sn ) semble converger.
e. Pour α = 0, 8, la suite (Sn ) semble diverger vers +∞.
f. Pour α = 20, la suite (Sn ) semble converger.

Document sous licence libre Creative Commons


16

Livre du professeur - Mathématiques Tle Spécialité - Chapitre 4 : Suites

g. Pour α = 10, la suite (Sn ) semble converger.


h. Pour α = 0, 1, la suite (Sn ) semble diverger vers +∞.

6. Il semble que la suite (Sn ) converge lorsque α > 1 et diverge vers +∞ lorsque α 6 1.

Pour aller plus loin


 
1 −α 1
1. Si α est un nombre négatif, alors α = k où −α est positif et la suite
k kα
est alors croissante. Ainsi, en ajoutant des valeurs de plus en plus grandes, il est
impossible que la somme converge.

2. a. Pour tout entier 2 6 k 6 n, on a 0 6 k 2 −√1 6 k 2 . Or, comme la fonction racine


carrée est croissante sur [0; +∞[, on a k 2 − 1 6 k. Et comme la fonction
1 1
inverse est décroissante sur ]0; +∞[, on a √ > .
k2 − 1 k
Pn 1
b. Pour tout entier n > 2, on a vn = α
avec α = 1. Or, on a vu précédemment
 n  k=2 k
P 1
que la suite α
semble diverger vers +∞.
k=2 k
c. Pour tout entier n > 2, un = 1 + vn donc un > vn . Par comparaison, on en
déduit que la suite (un ) diverge vers +∞.

Document sous licence libre Creative Commons


17

Livre du professeur - Mathématiques Tle Spécialité - Chapitre 4 : Suites

5.2 Corrigé du TP 2 : Approximation de π par la méthode d’Ar-


chimède
Question préliminaire
Le carré rouge est un carré de côté 1 (diamètre du cercle) donc T4 = 4. Le carré bleu est
un carré donc les diagonales
√ valent 1. Ainsi, d’après le théorème de Pythagore, la longueur
2 √
d’un côté du carré est et ainsi on a S4 = 2 2.
2

Méthode 1
1. On utilise les valeurs de T4 et S4 calculées dans les questions préliminaires pour
compléter le tableau (le tableau initial se trouve dans le dossier TICE).

2S4 T4
2. On utilise le fait que T8 = .
S4 + T4


3. On utilise le fait que S8 = S4 T8 .

4. On trouve une valeur approchée de π à 10−5 près, ce qui, pour l’époque d’Archimède,
est tout à fait extraordinaire.

Méthode 2
1. Les lignes 4 et 5 correspondent aux premiers termes des suites (T2n ) et (S2n ) calculés
dans la question préliminaire.

Document sous licence libre Creative Commons


18

Livre du professeur - Mathématiques Tle Spécialité - Chapitre 4 : Suites

2Sn Tn 2S2n−1 T2n−1


2. On utilise le fait que T2n = donc T2n = . On a également
√ Sn + Tn S2n−1 + T2n−1
S2n = S2n−1 T2n .

3. On doit saisir n = 9. On obtient alors l’encadrement suivant.

Pour aller plus loin


1. Pour tout entier 1 6 k < n, on note Ak le k-ième sommet du polygone bleu et O le
centre du polygone.
1
OAk Ak+1 est un triangle isocèle en O tel que OAk = OAk+1 = et on a :
2
−−→ −−−−→ 2π
OAk ; OAk+1 = .
n
Notons H le pied de la hauteur issue de O. OHAk est un triangle rectangle en H tel
1 −−→ −−→ π −−→ −−→ HAk π 
que OAk = et OAk ; OH = . On a sin OAk ; OH = d’où sin =
2 n OA k n
HAk 1 π  π 
et alors HAk = sin . On en déduit alors que Ak Ak+1 = sin .
1 2 n n
2
Pour tout entier 1 6 k < n, on note Bk le k-ième sommet du polygone rouge et O
le centre du polygone.
−−→ −−−−→ 2π
OBk Bk+1 est un triangle isocèle en O tel que OBk ; OBk+1 = .
n
Notons P le pied de la hauteur issue de O. OP Bk est un triangle rectangle en P tel
1 −−→ −→ π −−→ −→ P Bk π 
que OP = et OBk ; OP = . On a tan OBk ; OP = d’où tan =
2 n OP n
P Bk 1  π   π 
et alors P Bk = tan . On en déduit alors que Bk Bk+1 = tan . Enfin,
1 2 n n
2 π 
A1 A2 . . . An est un polygone régulier à n côtés de longueur sin d’où Sn =
π  n π 
n sin . Et B1 B2 . . . Bn est un polygone régulier à n côtés de longueur tan
n π  n
d’où Tn = n tan .
n
Document sous licence libre Creative Commons
19

Livre du professeur - Mathématiques Tle Spécialité - Chapitre 4 : Suites

π  π 
2Sn Tn 2 × n sin × n tan
2. On a =  π n  π n
Sn + Tn n sin + n tan
n n
π  π 
2n × sin × tan
=  π n  π n .
sin + tan
n n
π 
 π  sin
Or, tan = nπ .
n cos
n
π 
 π  sin
2n × sin × n π
n cos
2Sn Tn
D’où =  π n
Sn + Tn  π  sin
sin + nπ
n cos
n
π  π 
sin2 sin
2n ×  πn 2n × nπ π 
cos cos 2n × sin
=   πn  = π  n = π  n
 π  cos +1 cos +1 cos +1
sin n   n  n
π π

n cos cos
n n
π 
sin θ
= 2n nπ  , car cos(θ) + 1 = 2cos2 ( ).
2 cos2 2
2n
π
π  π π  π  sin
3. a. On a Sn T2n = n sin × 2n tan = 2n2 × 2 sin cos  2n 
n 2n 2n 2n cos π
2n

π r π π
2 2 2 2
= 4n sin . D’où Sn T2n = 4n sin = 2n sin . Or, puisque
2n      2n 2n
θ θ θ sin(θ)
sin(θ) = 2 sin cos , on a sin =   . On en déduit alors
2 2 2 θ
2 cos
π  2
√ sin
que Sn T2n = n n π .
cos
2n  
π
2Sn Tn sin
b. On a = 2n nπ 
Sn + Tn 2 cos2
π  π  2n
2 sin cos
= 2n 2n   2n , d’après la formule de l’énoncé
π
2 cos2
2n
Document sous licence libre Creative Commons
20

Livre du professeur - Mathématiques Tle Spécialité - Chapitre 4 : Suites

π
sin π
= 2n 2n
 π = 2n tan 2n = T2n .

cos
2n
√ π √
De plus, on a vu que Sn T2n = 2n sin d’où Sn T2n = S2n .
2n

Document sous licence libre Creative Commons


21

Livre du professeur - Mathématiques Tle Spécialité - Chapitre 4 : Suites

6 Travailler les automatismes


6.1 Exercices à l’oral
Corrigé exercice 17 :
Cette suite semble diverger vers −∞.

Corrigé exercice 18 :
Il semble que lim un = +∞ et lim vn = 3.
n→+∞ n→+∞

Corrigé exercice 19 :
La suite semble être majorée par 6.

Corrigé exercice 20 :
Cette suite semble converger vers 4.

6.2 Exercices
Corrigé exercice 21 :
1
1. a. On a wn ∈ ]1, 99; 2, 01[ ⇔ 1, 99 < 2 + √ < 2, 01
n
1
⇔ −0, 01 < √ < 0, 01
n
1
⇔ √ < 0, 01
n
1 1
⇔ √ < 0, 01 car √ > 0
n n
√ 1
⇔ n>
0, 01
⇔ n > 10 000
Ainsi, en prenant n0 = 10 001, on a bien wn ∈ ]1, 99; 2, 01[ pour tout entier
n > n0 .
1
b. On a |wn − 2| 6 10−4 ⇔ 2 + √ − 2 6 10−4
n
1
⇔ √ 6 10−4
n
1 1
⇔ √ 6 10−4 car √ > 0
n n
8
⇔ n > 10
Ainsi, en prenant n1 = 108 , on a bien |wn − 2| 6 10−4 pour tout entier n > n1 .
1
c. On a wn ∈ ]2 − ε; 2 + ε[ ⇔ 2 − ε < 2 + √ < 2 + ε
n
1
⇔ −ε < √ < ε
n
Document sous licence libre Creative Commons
22

Livre du professeur - Mathématiques Tle Spécialité - Chapitre 4 : Suites

1
⇔ √ <ε
n
1 1
⇔ √ < ε car √ > 0
n n
√ 1
⇔ n>
 ε2
1
⇔n>
ε
Ainsi,
 2 en posant n2 comme étant le plus petit entier strictement supérieur à
1
, on a bien wn ∈ ]2 − ε; 2 + ε[ pour tout entier n > n2 .
ε
2. On en déduit donc que lim wn = 2.
n→+∞

Corrigé exercice 22 :
104
1. a. On a un > 100 ⇔ 3n − 4 > 100 ⇔ 3n > 104 ⇔ n > .
3
104
Or, comme ≈ 34, 6, en prenant n0 = 35, on a bien un > 100 pour tout
3
entier n > n0 .
1004
b. On a un > 1000 ⇔ 3n − 4 > 1000 ⇔ 3n > 1004 ⇔ n > .
3
1004
Or, comme ≈ 334, 6, en prenant n1 = 335, on a bien un > 1000 pour tout
3
entier n > n1 .
A+4
c. On a un > A ⇔ 3n − 4 > A ⇔ 3n > A + 4 ⇔ n > .
3
Ainsi, en prenant comme valeur de n2 le plus petit entier supérieur ou égal à
A+4
, on a bien un > A pour tout entier n > n0 .
3
2. On en déduit que lim un = +∞.
n→+∞

Corrigé exercice 23 :
1. a. On a vn 6 −720 ⇔ −5n2 6 −720 ⇔ n2 > 144 ⇔ n > 12 car n est un entier
naturel.
Ainsi, en prenant n0 = 12, on a bien vn 6 −720 pour tout entier n > n0 .
b. On a vn 6 −3125 ⇔ −5n2 6 −3125 ⇔ n2 > 625 ⇔ n > 25 car n est un entier
naturel.
Ainsi, en prenant n1 = 25, on a bien vn 6 −3125 pour tout entier n > n1 .
r
−A −A
c. On a vn 6 A ⇔ −5n2 6 A ⇔ n2 > ⇔n> car n est un entier
5 5
naturel.
Ainsi,
r en prenant comme valeur de n2 le plus petit entier supérieur ou égal à
−A
, on a bien vn 6 A pour tout entier n > n2 .
5
2. On en déduit que lim vn = −∞.
n→+∞

Document sous licence libre Creative Commons


23

Livre du professeur - Mathématiques Tle Spécialité - Chapitre 4 : Suites

Corrigé exercice 24 :
2n + 3 2n + 3 − 2n − 4 −1
1. un 6 2 ⇔ 62⇔ 60⇔ 6 0. Or, pour tout entier
n+2 n+2 n+2
−1
naturel n, on a n + 2 > 0 donc 6 0. Ainsi, la suite (un ) est bien majorée par
n+2
2.

2. Pour tout entier n > 1, on a −1 6 (−1)n 6 1 ⇔ n2 − 1 6 n2 + (−1)n 6 n2 + 1. Par


1 n2 − 1 n2 + 1
ailleurs, > 0 donc 6 vn 6 . Or, pour tout entier n > 1, on a
n+1 n+1 n+1
n2 > 1 d’où n2 − 1 > 0 et donc 0 6 vn .

3. Pour tout entier naturel n, on a√ n 6 √n + 1. Comme √ la fonction


√ racine carrée
est croissante√sur [0; +∞[,
√ on√ a n 6√ n + 1. D’où n + 1 − n > 0. D’autre
( n + 1 − n)( n + 1 + n) n+1−n 1
part, wn = √ √ = √ √ = √ √ . Or,
n+1+ n n+1+ n n+1+ n
√ √ 1
n + 1 + n > 1 d’où √ √ 6 1. Ainsi la suite (wn ) est bien bornée entre
n+1+ n
0 et 1.

4. Soit n ∈ N. On note Pn la proposition : « −3 6 tn 6 −1 ». On souhaite démontrer


que Pn est vraie pour tout n ∈ N.
√ √
Initialisation : t0 = − 2 et −3 6 − 2 6 −1 donc P0 est vraie.
Hérédité : On considère un entier naturel k quelconque tel que Pk est vraie (hypo-
thèse de récurrence), autrement dit tel que −3 6 tk 6 −1. On souhaite démontrer
que Pk+1 est vraie, autrement dit que −3 6 tk+1 6 −1.
Par hypothèse de récurrence, on a −3 6 tk 6 −1.
3 1 1 11 1 9
D’où − 6 tk 6 − donc − 6 tk − 2 6 − ⇔ −3 6 tk+1 6 −1.
4 4 4 4 4 4
Ainsi, P0 est vraie et, pour tout entier k, lorsque Pk est vraie, alors Pk+1 est vraie
aussi. Par le principe de récurrence, on en déduit que, pour tout n ∈ N, Pn est vraie
donc −3 6 tn 6 −1.

Corrigé exercice 25 :
√ √
1. On a lim n = +∞ et lim n2 = +∞. D’où, par somme, lim n + n2 = +∞.
n→+∞ n→+∞ n→+∞

1 1
2. On a lim 4
= 0. D’où, par somme, lim 3 + 4 = 3.
n→+∞ n n→+∞ n
 n
4 4 1
3. Comme > 1 alors lim = +∞. De plus lim 2 = 0. D’où, par somme,
 3n n→+∞ 3 n→+∞ n
4 1
lim + 2 = +∞.
n→+∞ 3 n
1 1
4. On a lim √ = 0 et lim −n3 = −∞. D’où, par somme, lim √ − n3 = −∞.
n→+∞ n n→+∞ n→+∞ n

Document sous licence libre Creative Commons


24

Livre du professeur - Mathématiques Tle Spécialité - Chapitre 4 : Suites

5. Comme π > 1 alors lim π n = +∞. De plus lim n = +∞.


n→+∞ n→+∞

D’où, par somme, lim n + π = +∞. Donc lim −(n + π n ) = −∞.


n
n→+∞ n→+∞
 n
7 7
6. Comme −1 < < 1 alors lim = 0. De plus lim n5 = +∞.
10 n→+∞ 10 n→+∞
 n
7
D’où, par somme, lim −4 + + n5 = +∞.
n→+∞ 10

Corrigé exercice 26 :
1. On a lim n2 = +∞ et lim −n = −∞. On obtient donc une forme indéterminée.
n→+∞ n→+∞
 
2 2 1 1
Mais, pour tout entier naturel n > 1, on a rn = n −n = n 1 − . Or, lim =
n n→+∞ n
1
0. D’où par somme, on a lim 1+ = 1. De plus lim n2 = +∞. Ainsi, par produit,
n→+∞ n n→+∞
on a lim rn = +∞.
n→+∞


 
1
2. Pour tout entier naturel n > 1, on a sn = −n + n = −n 1 − √ .
n
1 1
Or, lim √ = 0. D’où par somme, on a lim 1 − √ = 1.
n→+∞ n n→+∞ n
De plus lim −n = −∞. Ainsi, par produit, on a lim sn = −∞.
n→+∞ n→+∞


 
7 7 1
3. Pour tout entier naturel n > 1, on a tn = −(− n + n ) = −n − √ +1 .
n6 n
√ √
Or, lim n = +∞ et lim n6 = +∞. D’où, par produit, lim n6 n = +∞. On
n→+∞ n→+∞ n→+∞
1
en déduit alors, par quotient, qu’on a lim 6 √ = 0. Et donc, par somme, on a
n→+∞ n n
1
lim − 6 √ + 1 = 1. Ainsi, lim tn = lim −n7 = −∞.
n→+∞ n n n→+∞ n→+∞
 
6 3 6 3 6 1
4. Pour tout entier naturel n > 1, on a un = −(n − n ) = −n + n = −n 1 − 3 .
n
1 1
Or, lim − 3 = 0. D’où, par somme, on a lim 1 − 3 = 1.
n→+∞ n n→+∞ n
6
On a lim −n = −∞. Ainsi, par produit, on a lim un = −∞.
n→+∞ n→+∞
 
5 3 1 1 5
5. Pour tout entier naturel n > 1, on a vn = n − n + n = n 1 − 2 + 4 .
n n
1 1 1 1
Or, lim − 2 = 0 et lim − 4 = 0. D’où, par somme, lim 1 − 2 + 4 = 1.
n→+∞ n n→+∞ n n→+∞ n n
5
On a lim n = +∞. Ainsi, par produit, on a lim vn = +∞.
n→+∞ n→+∞

Document sous licence libre Creative Commons


25

Livre du professeur - Mathématiques Tle Spécialité - Chapitre 4 : Suites

 
6 4 2 6 1 1 1
6. Pour tout entier naturel n > 1, on a wn = n −n +n −n = n 1− 2 + 4 − 5 .
n n n
1 1 1
Or, lim − 2 = 0, lim − 4 = 0 et lim − 5 = 0. D’où, par somme, on a
n→+∞ n n→+∞ n n→+∞ n
1 1 1 6
lim 1 − 2 + 4 − 5 = 1. On a lim n = +∞. Ainsi, par produit, on a
n→+∞ n n n n→+∞
lim wn = +∞.
n→+∞

Corrigé exercice 27 :
1. Par somme, on a lim n5 + 4 = +∞ et lim n − 3 = +∞.
n→+∞ n→+∞
5
D’où, par produit, lim (n + 4)(n − 3) = +∞.
n→+∞
 n
185 185
2. Comme −1 < < 1, alors lim = 0.
192 n→+∞ 192
 n
185
D’où, par produit, lim 5 × = 0.
n→+∞ 192
1
3. Par somme, on a lim 8n − 2 = +∞ et lim 3 + √ = 3.
n→+∞ n→+∞ n
 
1
D’où, par produit, lim (8n − 2) 3 + √ = +∞.
n→+∞ n
 n
144 144
4. Comme > 1, alors lim = +∞.
121 n→+∞ 121
 n
144
D’où, par produit, lim −2 × = −∞.
n→+∞ 121
1
5. Par somme, on a lim 6 − n4 = −∞ et lim 3 + 7 = 7.
n→+∞ n→+∞ n
 
4 1
D’où, par produit, lim (6 − n ) + 7 = −∞.
n→+∞ n3
6. Par somme, on a lim 4 − n7 = −∞ et lim n9 + 1 = +∞.
n→+∞ n→+∞
7 9
D’où, par produit, lim (4 − n )(n + 1) = −∞.
n→+∞

Corrigé exercice 28 :
1
1. On a lim = 0 et, par somme, lim 2n + 4 = +∞.
n→+∞ n2 n→+∞

On ne peut pas conclure directement.


1 2n + 4 2 4
Pour tout entier naturel n > 1, on a 2 (2n + 4) = 2
= + 2.
n n n n
2 4 2 4
Or, lim = 0 et lim 2 = 0. D’où, par somme, on a lim + 2 = 0.
n→+∞ n n→+∞ n n→+∞ n n

Document sous licence libre Creative Commons


26

Livre du professeur - Mathématiques Tle Spécialité - Chapitre 4 : Suites

1 2n + 4 4
2. Pour tout entier naturel n > 1, on a
(2n + 4) = =2+ .
n n n
4 4
Or, lim = 0. D’où, par somme, on a lim 2 + = 2.
n→+∞ n n→+∞ n
1 3 2 n3 + 2n2 1 2
3. Pour tout entier naturel n > 1, on a 4
(n + 2n ) = 4
= + 2.
n n n n
1 2 1 2
Or, lim = 0 et lim 2 = 0. D’où, par somme, on a lim + 2 = 0.
n→+∞ n n→+∞ n n→+∞ n n

1 √ −n − n √
4. Pour tout entier naturel n > 1, on a √ (−n − n) = √ = − n − 1.
n n
√ √
Or, lim − n = −∞. D’où, par somme, on a lim − n − 1 = −∞.
n→+∞ n→+∞

Corrigé exercice 29 :
1
1. On a lim = 0 et par somme, lim −8 − n6 = −∞.
n→+∞ n5 n→+∞

Donc on obtient une forme indéterminée. Mais, pour tout entier naturel n > 1, on
1 −8 − n6 8 8
a 5 (−8 − n6 ) = 5
= − 5 − n. Or, lim − 5 = 0 et lim −n = −∞.
n n n n→+∞ n n→+∞
8
D’où, par somme, on a lim − 5 − n = −∞.
n→+∞ n
1 −n3 + 3 3
(−n3 + 3) =
2. Pour tout entier naturel n > 1, on a = −n2 + .
n n n
3 3
Or, lim −n2 = −∞ et lim = 0. D’où, par somme, lim −n2 + = −∞.
n→+∞ n→+∞ n n→+∞ n
1 2n − 3 √ 3
3. Pour tout entier naturel n > 1, on a √ (2n − 3) = √ =2 n− √ .
n n n
√ 3 √ 3
Or, lim 2 n = +∞ et lim − √ = 0. D’où, par somme, lim 2 n − √ =
n→+∞ n→+∞ n n→+∞ n
+∞.
1 3 1 − n3 1
4. Pour tout entier naturel n > 1, on a
3
(1 − n ) = 3
= 3 − 1.
n n n
1 1
Or, lim 3 = 0. D’où, par somme, on a lim 3 − 1 = −1.
n→+∞ n n→+∞ n

Corrigé exercice 30 :
2
1. Par somme, on a lim n2 − 4 = +∞. D’où, par quotient, on a lim = 0.
n→+∞ n→+∞ n2 −4
1
2. On a lim n3 = +∞ et, par somme, lim 4 + = 4. D’où, par quotient, on a
n→+∞ n→+∞ n
3
n
lim = +∞.
n→+∞ 1
4+
n

Document sous licence libre Creative Commons


27

Livre du professeur - Mathématiques Tle Spécialité - Chapitre 4 : Suites

1 1
3. Par somme, on a lim 4 − √ = 4 et lim 3 − 6 = −6.
n→+∞ n n→+∞ n

1
4− √
n 4 2
D’où, par quotient, on a lim = =− .
n→+∞ 1 −6 3
3
−6
n
 n
4 4 1
4. Comme > 1, alors lim = +∞. D’où, par quotient, on a lim  n = 0.
3 n→+∞ 3 n→+∞ 4
3
 n
3 3
5. On a lim −n = −∞ et comme −1 < < 1, alors lim = 0.
n→+∞ 4 n→+∞ 4
−n
D’où, par quotient, on a lim  n = −∞.
n→+∞ 3
4
 n
5 5 1
6. Comme −1 < < 1, alors lim = 0. Par somme, on a lim 2 − 9 = −9.
7 n→+∞ 7 n→+∞ n
 n
5
7
D’où, par quotient, on a lim = 0.
n→+∞ 1
− 9
n2

Corrigé exercice 31 :
1. Par somme, on a lim 3n2 +4 = +∞ et lim 2n+2 = +∞. On ne peut pas conclure
n→+∞ n→+∞  
2 4
n 3+ 2
3n2 + 4 n
directement. Mais, pour tout entier naturel n, on a =   =
2n + 2 2
n 2+
  n
4
n 3+ 2
n 4 2
. Or, par somme, lim 3 + 2 = 3 et lim 2 + = 2. Ainsi, par
2 n→+∞ n n→+∞ n
2+
n  
4
  n 3+ 2
4 n
produit, lim n 3 + 2 = +∞. Donc, par quotient, lim = +∞.
n→+∞ n n→+∞ 2
2+
n
 n  
7 7 n  
n
5 5 21
2. Pour tout entier naturel n, on a  n =   = .
4 4 20
3 3
 n
21 21
Or, comme > 1, on a lim = +∞.
20 n→+∞ 20

Document sous licence libre Creative Commons


28

Livre du professeur - Mathématiques Tle Spécialité - Chapitre 4 : Suites

 
4 4
n 2− 2−
2n − 4 n n.
3. Pour tout entier naturel n, on a =  =
7 − 3n 7 7
n −3 −3
n n
4 7
Or, par somme, lim 2 − = 2 et lim − 3 = −3.
n→+∞ n n→+∞ n
4
2−
Donc, par quotient, lim n = 2 = −2.
n→+∞ 7 −3 3
−3
n
12n2 12
4. Pour tout entier naturel n > 1, on a 7
= 5 . Or, lim 5n5 = +∞.
5n 5n n→+∞
12
Donc, par quotient, lim = 0.
n→+∞ 5n5
 n  
3 3 n  
n
5 2
5. Pour tout entier naturel n, on a  n =  5  = .
 
2 2 5
3 3
 n
2 2
Or, comme −1 < < 1, on a lim = 0.
5 n→+∞ 5
n2 − 1 (n + 1)(n − 1)
6. Pour tout entier naturel n, on a = = n − 1.
n+1 n+1
Donc, par somme, lim n − 1 = +∞.
n→+∞

Corrigé exercice 32 :
1. Par somme, on a lim 4 − 5n = −∞ et lim 2n2 + 1 = +∞. On ne peut pas
n→+∞ n→+∞  
4
n −5
4 − 5n n
conclure directement. Pour tout entier naturel n, on a =   =
2n2 + 1 2
1
n 2+ 2
n
4
−5
n .
1
n 2+ 2
n
4 1
Or, par somme, lim − 5 = −5 et lim 2 + 2 = 2. Ainsi, par produit,
n→+∞ n n→+∞ n
4

1
 −5
lim n 2 + 2 = +∞. Donc, par quotient, lim n  = 0.
n→+∞ n n→+∞ 1
n 2+ 2
n

Document sous licence libre Creative Commons


29

Livre du professeur - Mathématiques Tle Spécialité - Chapitre 4 : Suites

 n  
6 6 n  
n
19 33
2. Pour tout entier naturel n, on a  n =  19  = .
 
4 4 38
11 11
 n
33 33
Or, comme −1 < < 1, on a lim = 0.
38 n→+∞ 38
 
2 3 3
n −6 + 2 −6 + 2
−6n2 + 3 n n .
3. Pour tout entier naturel n > 1, on a =  =
1

−n − 2n2 1
n2 − − 2 − −2
n n
3 1
Or, par somme, lim −6 + 2 = −6 et lim − − 2 = −2.
n→+∞ n n→+∞ n
3
−6 + 2
Donc, par quotient, lim n = −6 = 3.
n→+∞ 1 −2
− −2
n
 n  
18 18 n  
n
5 99
4. Pour tout entier naturel n, on a  n =  5  = .
 
16 16 40
11 11
 n
99 99
Or, comme > 1, on a lim = +∞.
40 n→+∞ 40
 
2 1 1 1 1
n 1+ + 2 1+ + 2
n2 + n + 1 n n n n .
5. Pour tout entier naturel n > 1, on a = =
n3 n3 n
1 1
Or, par somme, lim 1 + + 2 = 1 et lim n = +∞.
n→+∞ n n n→+∞
1 1
1+ + 2
Donc, par quotient, lim n n = 0.
n→+∞ n
n2 − 4 (n + 2)(n − 2)
6. Pour tout entier naturel n, on a = = n − 2.
n+2 n+2
Donc, par somme, lim n − 2 = +∞.
n→+∞

Corrigé exercice 33 :
1. On a lim n2 + 1 = +∞ donc, par théorème de comparaison, on a lim un = +∞.
n→+∞ n→+∞

2. On a lim −3n−4 = −∞ donc, par théorème de comparaison, on a lim vn = −∞.


n→+∞ n→+∞

3. On a lim −1+2n = +∞ donc par théorème de comparaison, on a lim wn = +∞.


n→+∞ n→+∞

Document sous licence libre Creative Commons


30

Livre du professeur - Mathématiques Tle Spécialité - Chapitre 4 : Suites

Corrigé exercice 34 :
Pour tout entier naturel n, on a√n3 6 n√3
+1. Comme la fonction
√ racine
√ carrée est croissante
sur [0; +∞[, on en déduit que n√ 6 n + 1 c’est-à-dire n n 6 n3 √
3 3 + 1.
Or, on a lim n = +∞ et lim n = +∞ donc, par produit, lim n n = +∞.
n→+∞ n→+∞ √ n→+∞
Ainsi, par théorème de comparaison, on a lim 3
n + 1 = +∞.
n→+∞

Corrigé exercice 35 :
1. Soit n > 2 un entier. On note Pn la proposition : « tn 6 −n2 ». On souhaite
démontrer que Pn est vraie pour tout n ∈ N.
Initialisation : Pour n = 2.
On a t0 = 5, t1 = t0 − 5 × 0 − 4 = 1 et t2 = t1 − 5 × 1 − 4 = −8.
On a t2 = −8 et −22 = −4 donc t2 6 −22 . On en déduit que P2 est vraie.
Hérédité : On considère un entier naturel k > 2 quelconque tel que Pk est vraie
(hypothèse de récurrence), autrement dit tel que tk 6 −k 2 . On souhaite démontrer
que Pk+1 est vraie, autrement dit que tk+1 6 −(k + 1)2 .
Par hypothèse de récurrence, on a tk 6 −k 2 .
D’où tk − 5k − 4 6 −k 2 − 5k − 4.
Or, −(k + 1)2 = −(k 2 + 2k + 1) = −k 2 − 2k − 1 donc −k 2 − 5k − 4 6 −(k + 1)2 .
Donc tk+1 6 −(k + 1)2 .
Ainsi, P2 est vraie et, pour tout entier k > 2, lorsque Pk est vraie, alors Pk+1 est
vraie aussi. Par le principe de récurrence, on en déduit que, pour tout entier n > 2,
Pn est vraie donc tn 6 −n2 .
2. On a lim −n2 = −∞ donc, par théorème de comparaison, on a lim tn = −∞.
n→+∞ n→+∞

Corrigé exercice 36 :
1. Pour tout entier naturel n, on a −1 6 (−1)n 6 1.
D’où 4 6 5 + (−1)n 6 6 et donc 4 × 0, 59n 6 un 6 6 × 0, 59n .
2. Comme −1 < 0, 59 < 1, on a lim 0, 59n = 0.
n→+∞
D’où, par produit, lim 4 × 0, 59 = 0 et lim 6 × 0, 59n = 0.
n
n→+∞ n→+∞
Ainsi, d’après le théorème des gendarmes, on a lim un = 0.
n→+∞

Corrigé exercice 37 :
1. Pour tout entier naturel n non nul, on a −1 6 sin(n) 6 1.
−1 1
D’où 3 6 vn 6 3 .
n n
−1 1
2. On a lim 3
= 0 et lim 3 = 0.
n→+∞ n n→+∞ n
Ainsi, d’après le théorème des gendarmes, on a lim vn = 0.
n→+∞

Document sous licence libre Creative Commons


31

Livre du professeur - Mathématiques Tle Spécialité - Chapitre 4 : Suites

Corrigé exercice 38 :
1
un = 4 +
n
1
vn = 4 − 2
n
4n + 2
wn =
n−5
2n + 3 7 + 2n2
rn = ×
n − 1 n n2 − 4
sn = 3 × 0, 52 + 4

Corrigé exercice 39 :
1
Soient (un ), (vn ) et (wn ) trois suites définies pour tout entier naturel n par un = −5 − 2 ,
n
1 1
vn = −5 + et wn = −5 + 3 . Déterminer la limite de la suite (wn ).
n n
Pour tout entier naturel n, on a un 6 wn 6 vn . On a lim un = lim vn = −5.
n→+∞ n→+∞
D’après le théorème des gendarmes, on a lim wn = −5.
n→+∞

Document sous licence libre Creative Commons


32

Livre du professeur - Mathématiques Tle Spécialité - Chapitre 4 : Suites

7 Exercices d’entraînement partie 1


Corrigé exercice 40 :
1. On a lim rn = 0.
n→+∞

2. On a lim sn = 0.
n→+∞

3. On a lim tn = 0.
n→+∞

7
4. Comme −1 < < 1 alors lim un = 0.
9 n→+∞

17
5. Comme −1 < < 1 alors lim vn = 0.
21 n→+∞

1
6. Comme n−7 = alors lim wn = 0.
n7 n→+∞

Corrigé exercice 41 :
On calcule les termes de la suite grâce à la calculatrice.


−2n4 + 3n n − 6
En prenant n0 = 19, on a bien ∈ ]−0, 334; −0, 333[ pour tout entier
6n4
n > n0 .

Corrigé exercice 42 :
On calcule les termes de la suite grâce à la calculatrice.

Document sous licence libre Creative Commons


33

Livre du professeur - Mathématiques Tle Spécialité - Chapitre 4 : Suites

3n2 − 8n + 12
En prenant n0 = 66, on a bien ∈ ]0, 12; 0, 13[ pour tout n > n0 .
24n2

Corrigé exercice 43 :
1. La suite (un ) semble converger vers 4.
2 4n2 − 2
2. Pour tout entier n > 1, on a 4 − = = un .
n2 n2
3. Soit ε > 0 un réel.
2
un ∈ ]4 − ε; 4 + ε[ ⇔ 4 − ε < 4 − <4+ε
n2
2
⇔ −ε < − <ε
n2
2
⇔ − <ε
n2
2
⇔0< 2 <ε
n
1 ε
⇔0< 2 <
n 2
2
⇔ n2 >
ε
r
2
⇔n>
ε
r
2
Ainsi, en prenant comme valeur de n0 le plus petit entier supérieur à , on a bien
ε
un ∈ ]4 − ε; 4 + ε[ pour tout entier n > n0 . Donc lim un = 4.
n→+∞

Corrigé exercice 44 :
7 −3(2n + 3) + 7
1. Pour tout entier naturel n, on a −3 + =
2n + 3 2n + 3
−6n − 9 + 7
=
2n + 3
−6n − 2
=
2n + 3
= vn .

2. Soit ε > 0 un réel.


7
vn ∈ ]−3 − ε; −3 + ε[ ⇔ −3 − ε < −3 + < −3 + ε
2n + 3
7
⇔ −ε < <ε
2n + 3
7
⇔ <ε
2n + 3
7
⇔ <ε
2n + 3

Document sous licence libre Creative Commons


34

Livre du professeur - Mathématiques Tle Spécialité - Chapitre 4 : Suites

1 ε
⇔ <
2n + 3 7
7
⇔ 2n + 3 >
ε
7
⇔ 2n > − 3
ε
7
−3
⇔n> ε
2
7
−3
Ainsi, en prenant comme valeur de n0 le plus petit entier supérieur à ε , on a
2
bien vn ∈ ]−3 − ε; −3 + ε[ pour tout n > n0 . Donc lim vn = −3.
n→+∞

Corrigé exercice 45 :
1. a. L’appel fonction(0,1) renvoie 39.
b. L’appel fonction(0,001) renvoie 499 481.

2. L’appel fonction(e) renvoie le plus petit entier N tel que |uN − 3| < e.

3. La suite (un ) semble converger vers 3.

Corrigé exercice 46 :
1. a. L’appel fonction(0,001) renvoie 11.
b. L’appel fonction(0,00001) renvoie 18.

2. L’appel fonction(e) renvoie le plus petit entier N tel que |wN − 1| < e.

3. La suite (wn ) semble converger vers 1.

Corrigé exercice 47 :
1. Cette affirmation est fausse. Prenons comme contre-exemple (un ) définie pour tout
entier naturel n par un = (−1)n . Cette suite est bien bornée entre −1 et 1 mais ne
converge pas.

2. Cette affirmation est vraie. Supposons par l’absurde que la suite (vn ) converge vers
` et n’est pas bornée. Soit ε > 0 un réel. Il n’existe alors aucun entier n0 tel que
vn ∈ ]` − ε; ` + ε[ pour tout n > n0 , car cette suite n’est pas bornée, ce qui contredit
le fait que lim vn = `.
n→+∞

3. Cette affirmation est fausse. Prenons comme contre-exemple (wn ) définie pour tout
1
entier n > 1 par wn = − . Cette suite est croissante et majorée par 10 mais elle ne
n
converge pas vers 10.

Document sous licence libre Creative Commons


35

Livre du professeur - Mathématiques Tle Spécialité - Chapitre 4 : Suites

Corrigé exercice 48 :
1. a. La formule saisie dans la cellule B3 est = 1/3*B2+2.
b. La suite (tn ) semble être décroissante.
c. La suite (tn ) semble converger vers 3.
2. Soit n ∈ N. On note Pn la proposition : « 0 < tn+1 < tn ». On souhaite démontrer
que Pn est vraie pour tout n ∈ N.
Initialisation : Pour n = 0.
1 11
t0 = 5 et t1 = t0 + 2 = donc 0 < t1 < t0 . On en déduit que P0 est vraie.
3 3
Hérédité : On considère un entier naturel k quelconque tel que Pk est vraie (hy-
pothèse de récurrence), autrement dit tel que 0 < tk+1 < tk . On souhaite démontrer
que Pk+1 est vraie, autrement dit que 0 < tk+2 < tk+1 .
Par hypothèse de récurrence, on a 0 < tk+1 < tk .
1 1 1 1
D’où 0 < tk+1 < tk . Et donc 2 < tk+1 + 2 < tk + 2 c’est-à-dire 0 < tk+2 < tk+1 .
3 3 3 3
Ainsi, P0 est vraie et, pour tout entier k, lorsque Pk est vraie, alors Pk+1 est vraie
aussi. Par le principe de récurrence, on en déduit que, pour tout n ∈ N, Pn est vraie
donc 0 < tn+1 < tn .
3. La suite (tn ) est décroissante et minorée par 0, elle converge donc vers un réel `.
1 1 2
4. On a ` = ` + 2 ⇔ ` − ` = 2 ⇔ ` = 2 ⇔ ` = 3.
3 3 3

Corrigé exercice 49 :
1. a. La formule saisie dans la cellule B3 est = B2/(RACINE(B2ˆ2+1)).
b. La suite (rn ) semble être décroissante.
c. La suite (rn ) semble converger vers 0.
2. Soit n ∈ N. On note Pn la proposition : « 0 < rn 6 1 ». On souhaite démontrer que
Pn est vraie pour tout n ∈ N.
Initialisation : Pour n = 0. On a r0 = 1 donc 0 < r0 6 1. On en déduit que P0 est
vraie.
Hérédité : On considère un entier naturel k quelconque tel que Pk est vraie (hy-
pothèse de récurrence), autrement dit tel que 0 < rk 6 1. On souhaite démontrer
que Pk+1 est vraie, autrement dit que 0 < rk+1 6 1.
Par hypothèse de récurrence, on a 0 < rk 6 1. Comme la fonction x 7→ x2 + 1 est
strictement croissante sur [0; +∞[, on a 02 + 1 < rk2 + 1 6 12 + 1 ⇔ 1 < rk2 + 1 6 2.
√ p
De
√ plus, la fonction
p racine
√ carrée est croissante sur [0; +∞[, d’où 1 < rk2 + 1 6
2 ⇔ 1 < rk2 + 1 6 2.
1
Comme la fonction inverse est décroissante sur ]0; +∞[, on a alors 1 > p 2 >
rk + 1
1
√ > 0.
2
Document sous licence libre Creative Commons
36

Livre du professeur - Mathématiques Tle Spécialité - Chapitre 4 : Suites

rk
Enfin, par hypothèse de récurrence, on a rk > 0 d’où rk > p 2 >0⇔1>
rk + 1
rk > rk+1 > 0, par hypothèse de récurrence.
Ainsi, P0 est vraie et, pour tout entier k, lorsque Pk est vraie, alors Pk+1 est vraie
aussi. Par le principe de récurrence, on en déduit que, pour tout n ∈ N, Pn est vraie
donc 0 < rn 6 1.

3. D’après la question précédente, pour tout entier naturel n on a rn+1 < rn donc la
suite (rn ) est décroissante.

4. Comme la suite (rn ) est décroissante et minorée, alors elle converge.


` √
5. ` = √ ⇔ `( `2 + 1 − 1) = 0
`2 + 1

⇔ ` = 0 ou `2 + 1 − 1 = 0
⇔ ` = 0 ou `2 + 1 = 1
⇔ ` = 0 ou ` = 0
Donc la suite (rn ) converge vers 0.

Corrigé exercice 50 :
n+1
P 1 Pn 1 1
1. Pour tout entier n > 1, un+1 − un = 3
− 3
= .
k=1 k k=1 k (n + 1)3
1
Or > 0, on en déduit que la suite (un ) est croissante.
(n + 1)3
1 1 n+1−n 1
2. a. Pour tout entier n > 1, − = = .
n n+1 n(n + 1) n(n + 1)
1 1 1 1 1 1
D’où, − + 6 − ⇔ − >
n (n + 1)3 n+1 n n+1 (n + 1)3
1 1
⇔ >
n(n + 1) (n + 1)3
Or, pour tout entier n > 1, n(n + 1) 6 (n + 1)2 6 (n + 1)3 .
1 1 1
D’où > 2
> .
n(n + 1) (n + 1) (n + 1)3
1 1 1
Ainsi, on a bien − + 3
6− .
n (n + 1) n+1
1
b. Soit n ∈ N∗ . On note Pn la proposition : « un 6 2− ». On souhaite démontrer
n
que Pn est vraie pour tout n ∈ N∗ .
Initialisation : Pour n = 1.
1 1 1
u1 = 3 = 1 et 2 − = 1 donc u1 6 2 − . On en déduit que P1 est vraie.
1 1 1
Hérédité : On considère un entier naturel k > 1 quelconque tel que Pk est
1
vraie (hypothèse de récurrence), autrement dit tel que uk 6 2 − . On souhaite
k
1
démontrer que Pk+1 est vraie, autrement dit que uk+1 6 2 − .
k+1

Document sous licence libre Creative Commons


37

Livre du professeur - Mathématiques Tle Spécialité - Chapitre 4 : Suites

1 1 1
Par hypothèse de récurrence, on a uk 6 2 − ⇔ uk + 3
6 2− +
k (k + 1) k
1
.
(k + 1)3
1 1 1
Or, d’après la question 2.a., 2 − + 3
62− .
k (k + 1) k+1
1
D’où uk+1 6 2 − .
k+1
Ainsi, P1 est vraie et, pour un entier k > 1, Pk est vraie, donc Pk+1 est vraie
aussi. Par le principe de récurrence, on en déduit que, pour tout n ∈ N∗ , Pn
1
est vraie donc un 6 2 − .
n
1
3. Pour tout entier n > 1, on a un 6 2 − . De plus, la suite (vn ) définie pour tout
n
1
n > 1 par 2 − est strictement croissante et de limite 2 lorsque n tend vers +∞.
n
Donc, pour tout n > 1, (un ) 6 vn 6 2. Ainsi, la suite (un ) est croissante et majorée
par 2 donc elle converge vers un réel `.

Corrigé exercice 51 :
1. En 2015, cette personne aura 2000 × 1, 03 + 150 = 2210 e sur son compte épargne.
En 2016, elle aura 2210 × 1, 03 + 150 = 2426, 3 e sur son compte épargne.
2. D’une année à l’autre, le solde disponible sur le compte épargne augmente de 3 %
grâce aux intérêts. Il est donc multiplié par 1,03. De plus, chaque année la personne
dépose 150 e sur son compte épargne, on ajoute donc 150 au solde disponible. Ainsi,
pour tout entier naturel n, on a un+1 = 1, 03un + 150.
3. Pour tout entier naturel n, on a vn+1 = un+1 + 5000
= 1, 03un + 150 + 5000
= 1, 03un + 5150
 
5150
= 1, 03 un +
1, 03
= 1, 03(un + 5000)
= 1, 03vn .
Donc (vn ) est une suite géométrique de raison q = 1, 03 et de premier terme v0 =
u0 + 5000 = 7000.
4. Pour tout entier naturel n, on a vn = v0 × q n = 7000 × 1, 03n .
D’où un = vn − 5000 = 7000 × 1, 03n − 5000.
5. Comme 1, 03 > 1, on a lim 1, 03n = +∞. D’où, par produit, lim 7000 × 1, 03n =
n→+∞ n→+∞
+∞. Ainsi, par somme, on a lim un = lim 7000 × 1, 03n − 5000 = +∞.
n→+∞ n→+∞

6. a. L’appel programme(4000) renvoie 9.


b. Au bout de 9 ans, le solde disponible sur le compte épargne dépassera les
4000 e.

Document sous licence libre Creative Commons


38

Livre du professeur - Mathématiques Tle Spécialité - Chapitre 4 : Suites

8 Exercices d’entraînement partie 2


Corrigé exercice 52 :
1. On a lim rn = +∞.
n→+∞

2. Comme lim n5 = +∞ alors lim sn = −∞.


n→+∞ n→+∞

3. On a lim tn = +∞.
n→+∞

4. Comme lim n = +∞ alors lim un = −∞.


n→+∞ n→+∞

16
5. Comme > 1 alors lim vn = +∞.
9 n→+∞

4
6. Comme > 1 alors lim wn = +∞.
3 n→+∞

Corrigé exercice 53 :
On obtient le tableau de valeurs suivant à la calculatrice.

En prenant n0 = 28, on a 3n2 − 81n + 50 > 100 pour tout n > n0 .

Corrigé exercice 54 :
On calcule les termes de la suite grâce à la calculatrice.

En prenant n0 = 12, on a en > 106 pour tout n > n0 .

Document sous licence libre Creative Commons


39

Livre du professeur - Mathématiques Tle Spécialité - Chapitre 4 : Suites

Corrigé exercice 55 :

2
√ √ A+4
1. un > A ⇔ (3n − 4) > A ⇔ 3n − 4 > A ⇔ 3n > A+4⇔n>
3

A+4
2. En prenant comme valeur de n0 le plus petit entier supérieur ou égal à , on
3
a un > A pour tout n > n0 . D’où lim un = +∞.
n→+∞

Corrigé exercice 56 :
3n2 + 1
1. vn > A ⇔ > A ⇔ 3n2 + 1 > 2An ⇔ 3n2 − 2 × A × n + 1 > 0
2n
√ √
2. 4A2 − 12 = 4(A2 − 3) = 4(A − 3)(A + 3).


Si A > 3 alors 4A2 − 12 > 0.

Si 0 < A < 3 alors 4A2 − 12 < 0.

3. 3n2 −2An+1 est un polynôme du second degré de discriminant 2


√ ∆ = (−2A) −4×3×
2
1 = 4A −12. D’après la question précédente, si 0 < A 6 − 3 alors pour tout entier
n > 1, on a 3n2 − 2nA + 1 > 0 et donc vn > A. Dans cette
√ situation n0 = 1 convient.
√ 2A + 4A2 − 12
Si A > 3 alors 3n2 − 2nA + 1 > 0 lorsque n > . Ainsi en prenant
6 √
2A + 4A2 − 12
comme valeur de n0 le plus petit entier supérieur ou égal à =
√ 6
A + A2 − 3
, on a vn > A pour tout n > n0 .
3
4. On en déduit que lim vn = +∞.
n→+∞

Corrigé exercice 57 :
1. Soit n ∈ N. On note Pn la proposition : « (1+a)n > 1+an ». On souhaite démontrer
que Pn est vraie pour tout n ∈ N.
Initialisation : Pour n = 0.
(1 + a)0 = 1 et 1 + a × 0 = 1 donc (1 + a)0 > 1 + a × 0. P0 est vraie.
Hérédité : On considère un entier naturel k quelconque tel que Pk est vraie (hypo-
thèse de récurrence), autrement dit tel que (1+a)k > 1+ak. On souhaite démontrer
que Pk+1 est vraie, autrement dit que (1 + a)k+1 > 1 + a(k + 1).
On a (1 + a)k+1 = (1 + a) × (1 + a)k . Or, par hypothèse de récurrence, on a (1 + a)k >
1 + ak.

Document sous licence libre Creative Commons


40

Livre du professeur - Mathématiques Tle Spécialité - Chapitre 4 : Suites

D’où (1 + a) × (1 + a)k > (1 + a) × (1 + ak), car 1 + a > 0.


Or, (1 + a) × (1 + ak) = 1 + ak + a + a2 k = 1 + (k + 1)a + a2 k.
Comme k > 0 et a2 > 0, alors 1 + (k + 1)a + a2 k > 1 + (k + 1)a.
On a donc (1 + a)k+1 > 1 + (k + 1)a.
Ainsi, P0 est vraie et, pour tout entier k, lorsque Pk est vraie, alors Pk+1 est vraie
aussi. Par le principe de récurrence, on en déduit que, pour tout n ∈ N, Pn est vraie
donc (1 + a)n > 1 + an.
A−1
2. On a 1 + an > A ⇔ an > A − 1 ⇔ n > .
a
A−1
Ainsi, en prenant comme valeur de n0 le plus petit entier supérieur ou égal à ,
a
on a bien 1 + an > A pour tout n > n0 .

3. D’après l’inégalité de Bernoulli, (1 + a)n > 1 + an. D’où (1 + a)n > A pour tout
n > n0 , c’est-à-dire q n > A pour tout n > n0 . Donc on a bien lim q n = +∞.
n→+∞

Corrigé exercice 58 :
1. Pour tout réel A, il existe un entier n0 tel que pour tout entier n > n0 , on a un > A.

2. a. Comme lim un = +∞, alors pour tout réel A, il existe un entier n0 tel que
n→+∞
pour tout entier n > n0 , un > A. D’où, pour tout entier n > n0 , −un 6 −A.
b. Ainsi, en posant B = −A, il existe un entier n0 tel que pour tout n > n0 , on a
−un 6 B. Donc on a bien lim −un = −∞.
n→+∞

Corrigé exercice 59 :

1. On a un 6 A ⇔ − 5n + 7 6 A

⇔ 5n + 7 > −A
⇔ 5n + 7 > (−A)2
⇔ 5n + 7 > A2
⇔ 5n > A2 − 7
A2 − 7
⇔n>
5
A2 − 7
Ainsi, en prenant comme valeur de n0 le plus petit entier supérieur ou égal à ,
5
on a bien un 6 A pour tout n > n0 .

2. On en déduit que lim un = −∞.


n→+∞

Document sous licence libre Creative Commons


41

Livre du professeur - Mathématiques Tle Spécialité - Chapitre 4 : Suites

Corrigé exercice 60 :
1. a. L’appel fonction(10) renvoie 5.
b. L’appel fonction(50) renvoie 7.

2. L’appel fonction(p) renvoie le plus petit entier N tel que vN 6 −10P .

3. La suite (vn ) semble diverger vers −∞.

Corrigé exercice 61 :
1. L’affirmation est fausse. Prenons comme contre-exemple (un ) définie pour tout entier
naturel n par un = (−2)n . Cette suite n’est pas majorée mais n’a pas de limite.

2. L’affirmation est vraie. Supposons par l’absurde que la suite (vn ) est majorée par M .
Il n’existe alors aucun entier n0 tel que vn > M pour tout n > n0 , ce qui contredit
le fait que lim vn = +∞. C’est absurde. La suite ne peut donc pas être majorée.
n→+∞

1
3. L’affirmation est fausse. La suite (wn ) définie pour tout entier n > 1 par wn = 1 −
n
est strictement croissante et pourtant lim wn = 1.
n→+∞

4. L’affirmation est fausse. La suite (tn ) définie pour tout entier naturel n par tn =
n + 2 cos(n) a bien pour limite +∞ (par théorème de comparaison) mais n’est pas
croissante.

Corrigé exercice 62 :
1. L’appel fonction(2) renvoie 5. L’appel fonction(5) renvoie 8.

2. L’appel fonction(p) renvoie le plus petit entier N tel que zN > 10P .

3. La suite (zn ) semble diverger vers +∞.

Document sous licence libre Creative Commons


42

Livre du professeur - Mathématiques Tle Spécialité - Chapitre 4 : Suites

9 Exercices d’entraînement partie 3


Corrigé exercice 63 :
1 1
On a lim 7
= 0. D’où, par somme, lim 7 + 8 = 8.
n→+∞ n n→+∞ n

Corrigé exercice 64 :
On a lim −2n2 = −∞ et lim 3n = +∞. On ne peut donc pas conclure.
n→+∞ n→+∞  
2 2 3 4
Pour tout entier naturel n non nul, on a −2n + 3n + 4 = n −2 + + 2 . Or,
n n
3 4 3 4
lim = 0 et lim 2 = 0. Donc, par somme, lim −2 + + 2 = −2. Or, comme
n→+∞ n n→+∞ n n→+∞ n n √
lim n2 = +∞, alors, par produit, lim −2n2 +3n+4 = −∞. Aussi, comme lim n=
n→+∞ √ n→+∞ n→+∞
+∞ alors, par produit, lim n(−2n2 + 3n + 4) = −∞.
n→+∞

Corrigé exercice 65 :
Par somme, on a lim −n2 − 3n + 1 = −∞ et lim 2n2 + 4 = +∞.
n→+∞ n→+∞
On ne peut donc pas conclure.  
3 21 3 1
n −1 − + 2 −1 − + 2
−n2 − 3n + 1 n n n n .
Pour tout entier naturel n non nul, on a = =
4
 
2n2 + 4 4
n2 2 + 2 2+ 2
n n
3 1 4
Or, par somme, lim −1 − + 2 = −1 et lim 2 + 2 = 2.
n→+∞ n n n→+∞ n
−n2 − 3n + 1 1
Ainsi, par quotient, lim =− .
n→+∞ 2n2 + 4 2

Corrigé exercice 66 :
1. a. On a lim un = 0 et lim vn = +∞.
n→+∞ n→+∞
1
b. Pour tout entier n > 1, un × vn = × n = 1.
n
c. Donc lim un × vn = 1
n→+∞

2. a. On a lim un = 0 et lim vn = +∞.


n→+∞ n→+∞
1
b. Pour tout entier n > 1, un × vn = × n2 = n.
n
c. Donc lim un × vn = +∞
n→+∞

3. Le produit d’une suite qui converge vers 0 et d’une suite qui diverge vers ±∞ est
une forme indéterminée car on ne peut pas conclure de façon générale quant à
sa convergence. Lorsqu’on obtient ce type de cas, on doit alors trouver une autre
méthode pour calculer la limite.

Document sous licence libre Creative Commons


43

Livre du professeur - Mathématiques Tle Spécialité - Chapitre 4 : Suites

Corrigé exercice 67 :
1. On a lim un = 0 et lim vn = 0.
n→+∞ n→+∞

1
un 2 1 n 1
2. a. Pour tout entier n > 1, = n = 2× = .
vn 1 n 1 n
n
un
b. Donc lim = 0.
n→+∞ vn

1
vn 1 n2
3. a. Pour tout entier n > 1, = n = × = n.
un 1 n 1
n2
vn
b. Donc lim = +∞.
n→+∞ un

4. Le quotient de deux suites qui convergent vers 0 est une forme indéterminée car on
ne peut pas conclure de façon générale quant à sa convergence. Lorsqu’on obtient
ce type de cas, on doit alors trouver une autre méthode pour calculer la limite.

Corrigé exercice 68 :
1. On a lim un = +∞ et lim vn = +∞.
n→+∞ n→+∞

un n2
2. a. Pour tout entier naturel n > 1, = = n.
vn n
un
b. Donc lim = +∞.
n→+∞ vn
vn n 1
3. a. Pour tout entier naturel n > 1, = 2 = .
un n n
vn
b. Donc lim = 0. 3. Le quotient de deux suites qui divergent vers ±∞ est
n→+∞ un
une forme indéterminée car on ne peut pas conclure de façon générale quant
à sa convergence. Lorsqu’on obtient ce type de cas, on doit alors trouver une
autre méthode pour calculer la limite.

Corrigé exercice 69 :
1. Si q = 0 alors (q n ) est la suite constante égale à 0. Ainsi lim q n = 0.
n→+∞

1
2. a. Si 0 < q < 1 alors > 1.
q
 n
1 1
b. Comme > 1 on a alors lim = +∞.
q n→+∞ q
1
c. Par quotient, on a lim  n = lim q n = 0.
n→+∞ 1 n→+∞

Document sous licence libre Creative Commons


44

Livre du professeur - Mathématiques Tle Spécialité - Chapitre 4 : Suites

 n
1 1
3. Si −1 < q < 0 alors − > 1 et donc lim − = +∞. D’où lim (−q)n = 0.
q n→+∞ q n→+∞
n n n n
Or, q = (−1) × (−q) et la suite ((−1) ) est bornée entre −1 et 1 donc, d’après
le théorème des gendarmes, lim q n = 0.
n→+∞

Corrigé exercice 70 :
1 k2 − 1 (k − 1)(k + 1)
1. a. Pour tout entier 2 6 k 6 n, on a 1 − 2
= 1
= .
k k k2
    
1 1 1
b. Ainsi, pour tout entier n > 2, on a un = 1 − 2 1 − 2 ... 1 − 2
2 3 n
1×3 2×4 (n − 1)(n + 1) 1 × (n + 1) n+1
= × 2 × ... × = = .
22 3 n2 2×n 2n
2. On a lim n + 1 = +∞ et lim 2n = +∞. On ne peut donc pas conclure.
n→+∞ n→+∞
 
1
n 1+  
n+1 n 1 1
Pour tout entier n > 2, on a un = = = 1+ .
2n n×2 2 n
1 1
Or lim = 0 d’où, par somme, lim 1 + = 1.
n→+∞ n n→+∞ n
 
1 1 1
Ainsi, par quotient, lim un = lim 1+ = .
n→+∞ n→+∞ 2 n 2

Corrigé exercice 71 :
1. a. Pour tout entier 1 6 k 6 n, on a
(2k 2 + 2k + 1) − (2k 2 − 2k + 1)
(2k 2 + 2k + 1)(2k 2 − 2k + 1)
2k 2 + 2k + 1 − 2k 2 + 2k − 1
= 4
4k − 4k 3 + 2k 2 + 4k 3 − 4k 2 + 2k + 2k 2 − 2k + 1
4k
= 4 .
4k + 1
4k 1 1
b. D’où 4 = 2 − 2 .
4k + 1 2k − 2k + 1 2k + 2k + 1
Ainsi, pour tout entier n > 1, on a :
Pn 4k
bn = 4
4k + 1
 k=1     
1 1 1 1 1
= 1− + − + ... + −
5 5 13 2n2 − 2n + 1 2n2 + 2n + 1
1
=1− 2 .
2n + 2n + 1
−1
2. Par somme, on a lim 2n2 +2n+1 = +∞. D’où, par quotient, lim =
n→+∞ n→+∞ 2n2 + 2n + 1
0.
1
Et donc, par somme à nouveau, lim bn = lim 1 − 2 = 1.
n→+∞ n→+∞ 2n + 2n + 1

Document sous licence libre Creative Commons


45

Livre du professeur - Mathématiques Tle Spécialité - Chapitre 4 : Suites

Corrigé exercice 72 :
1. a. Pour tout entier 1 6 k 6 n, on a
   
1 1 1 1 1 1 1 k + 1 − k 1 k + 2 − (k + 1)
− − − = × − ×
2 k k+1 2 k+1 k+2 2 k(k + 1) 2 (k + 1)(k + 2)
1 1
= −
2k(k + 1) 2(k + 1)(k + 2)
k+2−k
=
2k(k + 1)(k + 2)
2
=
2k(k + 1)(k + 2)
1
= .
k(k + 1)(k + 2)
b. Ainsi, pour tout entier n > 1, on a
Pn 1
cn =
k=1 k(k + 1)(k + 1)
n 1
   
P 1 1 1 1 1
= − − −
k=1 2 k k+1 2 k+1 k+2
   
1 1 1 1
= 1− − −
2 2 n+1 n+2
 
1 1 n + 2 − (n + 1)
= −
2 2 (n + 1)(n + 2)
1 1
= −
4 2(n + 1)(n + 2)
2. On a lim n + 1 = +∞ et lim n + 2 = +∞.
n→+∞ n→+∞

D’où, par produit, lim 2(n + 1)(n + 2) = +∞.


n→+∞
1
Ainsi, par inverse, lim = 0.
n→+∞ 2(n + 1)(n + 2)

1
Et donc, par somme, lim cn = .
n→+∞ 4

Corrigé exercice 73 :
1. Chaque année, 80 % des abonnés de l’année précédente renouvellent leur abon-
nement, le nombre d’abonnés de l’année précédente est donc multiplié par 0,8.
De plus, il y a 450 nouvelles inscriptions. Ainsi, pour tout entier naturel n, on
a an+1 = 0, 8an + 450.

2. a. Pour tout entier naturel n, on a bn+1 = an+1 − 2250


= 0, 8an + 450 − 2250
= 0, 8an − 1800
 
1800
= 0, 8 an −
0, 8
= 0, 8(an − 2250)
= 0, 8bn .

Document sous licence libre Creative Commons


46

Livre du professeur - Mathématiques Tle Spécialité - Chapitre 4 : Suites

Ainsi, (bn ) est une suite géométrique de raison q = 0, 8 et de premier terme


b0 = a0 − 2250 = 1250.
b. Pour tout entier naturel n, on a alors bn = b0 × q n = 1250 × 0, 8n .
c. On a donc an = bn + 2250 = 1250 × 0, 8n + 2250.
3. Comme −1 < 0, 8 < 1, on a lim 0, 8n = 0.
n→+∞
D’où, par produit, lim 1250 × 0, 8n = 0.
n→+∞
Ainsi, par somme, lim 1250 × 0, 8n + 2250 = 2250. Sur le long terme, on peut donc
n→+∞
supposer que le nombre d’inscriptions va se stabiliser autour de 2250.

Corrigé exercice 74 :
1. Chaque année, 90 % des oiseaux bagués l’année précédente reviennent dans la ré-
serve, le nombre d’oiseaux de l’année précédente est donc multiplié par 0,9. De plus,
il y a 100 nouveaux oiseaux qui rejoignent la réserve. Ainsi, pour tout entier naturel
n, on a un+1 = 0, 9un + 100.
2. a. Pour tout entier naturel n, on a vn+1 = un+1 − 1000
= 0, 9un + 100 − 1000
= 0, 9un − 900 
900
= 0, 9 un −
0, 9
= 0, 9(un − 1000)
= 0, 9vn .
Ainsi, (vn ) est une suite géométrique de raison q = 0, 9 et de premier terme
v0 = u0 − 1000 = −400.
b. Pour tout entier naturel n, on a alors vn = v0 × q n = −400 × 0, 9n .
D’où un = vn + 1000 = −400 × 0, 9n + 1000.
3. Comme −1 < 0, 9 < 1, on a lim 0, 9n = 0.
n→+∞
D’où, par produit, lim −400 × 0, 9n = 0.
n→+∞
Ainsi, par somme, lim −400 × 0, 9n + 1000 = 1000.
n→+∞
Au bout d’un très grand nombre d’années, le nombre d’oiseaux présents dans la
réserve va se stabiliser autour de 1000 individus.
4. a. On complète l’algorithme comme ci-dessous.

Document sous licence libre Creative Commons


47

Livre du professeur - Mathématiques Tle Spécialité - Chapitre 4 : Suites

b. On peut programmer cet algorithme en Python comme ci-dessous, par exemple.

L’appel oiseaux() renvoie la valeur 14. Ainsi, en 2019 + 14 = 2033, le nombre


d’oiseaux dans la réserve sera supérieur à 900.

Corrigé exercice 75 :
1
1. a. D’une étape à l’autre, on noircit de la surface verte restante.
9
b. Si à l’étape n, la surface noircie est An alors la surface verte restante est 1−An .
1
Or, comme à chaque étape on noircit de la surface verte restante on a, pour
9
1 1 1 8 1
tout entier n > 1, An+1 = An + (1 − An ) = An + − An = An + .
9 9 9 9 9
2. a. Pour tout entier n > 1, on a Bn+1 = An+1 − 1
8 1
= An + − 1
9 9
8 8
= An −
9 9
8
= (An − 1)
9
8
= Bn .
9
8
Donc (Bn ) est une suite géométrique de raison q = et de premier terme
9
8
B1 = A1 − 1 = − .
9
 n−1  n
n−1 8 8 8
b. Pour tout entier n > 1, on a Bn = B1 × q =− × =− .
9 9 9
 n
8
c. D’où An = Bn + 1 = − + 1.
9
 n
8 8
d. Comme −1 < < 1, alors lim = 0.
9 n→+∞ 9
 n
8
D’où, par somme, lim An = lim − + 1 = 1.
n→+∞ n→+∞ 9

Corrigé exercice 76 :
1. a. D’une étape à l’autre, un segment est partagé en 4 segments de même longueur.
b. Pour tout entier n > 1, on a donc cn+1 = 4cn . Donc (cn ) est une suite géomé-
trique de raison q = 4 et de premier terme c1 = 3.

Document sous licence libre Creative Commons


48

Livre du professeur - Mathématiques Tle Spécialité - Chapitre 4 : Suites

c. Pour tout entier n > 1, on a cn = c1 × q n−1 = 3 × 4n−1 .

2. a. À chaque étape, la longueur du segment précédent est divisée par 3. Ainsi,


1
pour tout entier n > 1, on a `n+1 = `n . Donc (`n ) est une suite géométrique
3
0 1
de raison q = et de premier terme `1 .
3
 n−1
0 n−1 1
b. Pour tout entier n > 1, on a alors `n = `1 × (q ) = `1 × .
3
 n−1
n−1 1
c. Pour tout entier n > 1, on a pn = cn × `n = 3 × 4 × `1 × =
3
 n−1
4
3`1 × .
3
 n−1
4 4
d. Comme > 1, on a alors lim = +∞. Or, comme `1 représente une
3 n→+∞ 3
longueur, on a 3`1 > 0 et donc lim pn = +∞.
n→+∞

3. Notons An l’aire du flocon à l’étape n.


Pour passer de l’étape n à l’étape n+1, on ajoute à l’aire précédente l’aire de 3×4n−1
triangles équilatéraux de côté `n+1 . Notons ABC un de ces triangles et H le projeté
orthogonal de A sur [BC]. ABH est un triangle rectangle en H donc d’après le
théorème  de  Pythagore, on a AB 2 = AH 2 + BH 2 d’où √ AH 2 = AB 2 − BH 2 =
2
`n+1 `2 3 3
`2n+1 − = `2n+1 − n+1 = `2n+1 et donc AH = `n+1 .
2 4 4 2

3 √
`n+1 × `n+1 3 2
Ainsi, l’aire du triangle ABC est 2 = ` .
2 4 n+1

n−1 3 2
On a donc An+1 = An + 3 × 4 × `
4 n+1
√   n 2 √   n 2
n−1 3 1 n−1 3 1
= An + 3 × 4 × × `1 × = An + 3 × 4 × × `1 × =
√ 4  n 3 √ 4 3
n−1 3 2 1 3 3 2 4n−1
An +3×4 × ×`1 × = An + ×`1 × n Soit n > 2 un entier. On note
4 9√ 4 9
4 3 × 42 3 × 4n−2
 
3 2 3 3 ×
Pn la proposition : « An = × `1 1 + 1 + 2 + + ... + ».
4 9 9 93 9n−1
On souhaite démontrer que Pn est vraie pour tout n > 2.
√ √ √ √
3 2 3 2 3 2 3
Initialisation : Pour n = 2. A2 = × `1 + 3 × × `2 = × `1 + 3 × ×
 2 √ √ 4 √  4  √4  4
1 3 3 1 2 3 1 3 3
`1 = × `21 + 3 × × `1 = × `21 1 + et × `21 1 + 1 =
√3 
4
 √
4

9

4 √ 3  4  9
3 3 3 1 3 3
× `21 1 + = × `21 1 + donc A2 = × `21 1 + 1 . On en déduit
4 9 4 3 4 9
que P2 est vraie.
Hérédité : On considère un entier naturel k > 2 quelconque tel que Pk est vraie
(hypothèse de récurrence), autrement dit tel que :

Document sous licence libre Creative Commons


49

Livre du professeur - Mathématiques Tle Spécialité - Chapitre 4 : Suites

√  2 k−2

3 3 3 × 4 3 × 4 3 × 4
Ak = × `21 1 + 1 + 2 + + ... + .
4 9 9 93 9k−1
On souhaite démontrer que Pk+1 est vraie, autrement dit que :

3 × 4 3 × 42 3 × 4k−1
 
3 2 3
Ak+1 = × `1 1 + 1 + 2 + + ... + .
4 9 9 93 9k

3 3 4k−1
On a Ak+1 = Ak + × `21 × k .
4 9
√  √
3 × 4 3 × 42 3 × 4k−2 3 3 2 4k−1

3 2 3
D’où Ak+1 = ×`1 1 + 1 + 2 + 3
+ . . . + k−1
+ ×`1 × k
√ 4 9 9 9 9 4 9
2 k−2 k−1
 
3 3 3×4 3×4 3×4 3×4
= × `21 1 + 1 + 2 + 3
+ ... + k−1
+ .
4 9 9 9 9 9k
Conclusion : Ainsi, P2 est vraie et, pour un entier k > 2, lorsque Pk est vraie, alors
Pk+1 est vraie aussi. Par√ le principe de récurrence, on en déduit que, pour tout  n > 2,
2 n−2
3 2 3 3×4 3×4 3×4
Pn est vraie donc An = ×`1 1 + 1 + 2 + 3
+ ... + . Ainsi,
4 √ 9  9 9 9n−1
3 × 4 3 × 42 3 × 4n−2

3 2 3
pour tout entier n > 2, on a An = ×`1 1 + 1 + 2 + + ... +
4 9 9  93 9n−1
 n−1 
4
√   2 n−2
 √ 1−
3 2 3 4 4 4 3 2  1 9 
= ×`1 1 + 1 + + 2 + . . . + n−2 = ×`1 1 + ×

4 9 9 9 9 4 3 4 
1−
 
9
  n−1 
4
√ 1− √  n−1 ! !
3  1 9  3 1 4 9
= × `21 1 + ×  = × `21 1 + × 1 − ×
 
4  3 5  4 3 9 5
9
√  n−1 !!  n−1
3 3 4 4 4
= × `21 1 + × 1 − . Or, −1 < < 1 d’où lim = 0.
4 5 9 9 n→+∞ 9
 n−1
4 3
Ainsi, par somme, on a lim 1 − = 1. D’où, par produit, lim ×
n→+∞ 9 n→+∞ 5
 n−1 !  n−1 !
4 3 3 4 8
1− = . Et, par somme, lim 1 + × 1 − = . Donc, par
9 5 n→+∞ 5 9 5
√  n−1 !! √
3 3 4 2 3 2
produit, lim An = lim × `21 1 + × 1 − = ` . Ainsi, la
n→+∞ n→+∞ 4 5 9 5 1

2 3 2
suite (An ) converge vers ` . Il est surprenant que cette suite converge car on
5 1
ajoute toujours de nouveaux triangles, on pourrait donc imaginer que cette suite
diverge vers +∞. La figure vers laquelle on tend a donc un périmètre qui diverge
mais une aire qui converge.

Document sous licence libre Creative Commons


50

Livre du professeur - Mathématiques Tle Spécialité - Chapitre 4 : Suites

10 Exercices d’entraînement partie 4


Corrigé exercice 77 :
Pour tout entier naturel n, on a −1 6 (−1)n 6 1 d’où n3 − 1 6 n3 + (−1)n 6 n3 + 1.
Or, lim n3 −1 = +∞. Donc, par théorème de comparaison, on a lim n3 +(−1)n = +∞.
n→+∞ n→+∞

Corrigé exercice 78 :
Pour tout entier naturel n, on a −1 6 cos(n) 6 1.
D’où −2n7 − 1 6 −2n7 + cos(n) 6 −2n7 + 1. Or, lim −2n7 + 1 = −∞.
n→+∞
Donc, par théorème de comparaison, on a lim −2n7 + cos(n) = −∞.
n→+∞

Corrigé exercice 79 :
1 sin(n) 1
Pour tout entier naturel n non nul, on a −1 6 sin(n) 6 1 d’où − √ 6 √ 6√ .
n n n
7 1 7 sin(n) 7 1 1 7
Ainsi, − √ 6 + √ 6 + √ . Or, lim √ = 0 d’où, par somme, lim −
3 n 3 n 3 n n→+∞ n n→+∞ 3
1 7 7 1 7
√ = et lim +√ = .
n 3 n→+∞ 3 n 3
7 sin(n) 7
Donc, d’après le théorème des gendarmes, lim + √ = .
n→+∞ 3 n 3

Corrigé exercice 80 :
1. Soit n > 5 un entier. On note Pn la proposition : « bn > n − 3 ». On souhaite
démontrer que Pn est vraie pour tout n > 5.
Initialisation : Pour n = 5.
1 553
b5 = b4 + 4 − 2 = et 5 − 3 = 2 donc b5 > 5 − 3. On en déduit que P5 est vraie.
3 243
Hérédité : On considère un entier naturel k > 5 quelconque tel que Pk est vraie
(hypothèse de récurrence), autrement dit tel que bk > k − 3. On souhaite démontrer
que Pk+1 est vraie, autrement dit que bk+1 > (k + 1) − 3.
1 1 1 1
Par hypothèse de récurrence, on a bk > k − 3 d’où bk > (k − 3) ⇔ bk > k − 1.
3 3 3 3
1 1 4
On a donc, bk + k − 2 > k − 1 + k − 2 ⇔ bk+1 > k − 3.
3 3 3
4 4 1
Or, k − 3 > k − 2 ⇔ k − 3 − (k − 2) > 0 ⇔ k − 1 > 0 ⇔ k > 3. Donc, comme
3 3 3
4
k > 5, on a bien k − 3 > k − 2 et ainsi bk+1 > (k + 1) − 3.
3
Ainsi, P5 est vraie et, pour un entier k > 5, lorsque Pk est vraie, alors Pk+1 est vraie
aussi. Par le principe de récurrence, on en déduit que, pour tout n > 5, Pn est vraie
donc bn > n − 3.

2. On a lim n − 3 = +∞ donc, par théorème de comparaison, lim bn = +∞.


n→+∞ n→+∞

Document sous licence libre Creative Commons


51

Livre du professeur - Mathématiques Tle Spécialité - Chapitre 4 : Suites

Corrigé exercice 81 :
1. La fonction x 7→ x2 − 3x + 5 est un trinôme du second degré dont le discriminant
∆ = 3√2 − 4 × 5 = −11 est négatif. Donc,
√ pour tout entier naturel n, n2 − 3n + 5 > 0
d’où n2 − 3n + 5 > 0. Ainsi, n4 + n2 − 3n + 5 − 8 > n4 − 8.

2. On a lim n4 − 8 = +∞ donc, par théorème de comparaison, lim cn = +∞.


n→+∞ n→+∞

Corrigé exercice 82 :
1. Pour tout entier n > 1, on a −1 6 sin(2n) 6 1 ⇔ −3 6 3 sin(2n) 6 3
⇔ −11 6 3 sin(2n) − 8 6 −5
1 1 1
⇔− > > − car la fonction inverse est décroissante sur ] − ∞; 0[
11 3 sin(2n) − 8 5
2 − 4n 2 − 4n 2 − 4n
⇔− 6 6− car, pour tout n > 1, 2 − 4n < 0
11 3 sin(2n) − 8 5
4n − 2 2 − 4n 4n − 2
⇔ 6 6
11 3 sin(2n) − 8 5
4n − 2 4n − 2
⇔ 6 dn 6 .
11 5
4n − 2
2. On a lim = +∞ donc, par théorème de comparaison, lim dn = +∞.
n→+∞ 11 n→+∞

Corrigé exercice 83 :
1. Pour tout entier n > 1, on a
−1 6 sin(2n) 6 1 ⇔ −4 6 4 sin(2n) 6 4
⇔ −11 6 4 sin(2n) − 7 6 −3
1 1 1
⇔− > >−
11 4 sin(2n) − 7 3
11n2 11n2 11n2
⇔− > >−
11 4 sin(2n) − 7 3
11n2 11n2
⇔ −n2 > >− . Pour tout entier n > 1, on a bien en 6 −n2 .
4 sin(2n) − 7 3
2. On a lim −n2 = −∞ donc, par théorème de comparaison, lim en = −∞.
n→+∞ n→+∞

Corrigé exercice 84 :
1. Pour tout entier naturel n, on a
cos(3n) = cos(2n) cos(n) − sin(2n) sin(n)
= cos2 (n) − sin2 (n) cos(n) − 2 sin2 (n) cos(n)
 

= cos(n) cos2 (n) − sin2 (n) − 2 sin2 (n)


 

= cos(n) cos2 (n) − 3 sin2 (n)


 

Document sous licence libre Creative Commons


52

Livre du professeur - Mathématiques Tle Spécialité - Chapitre 4 : Suites

= cos(n) 4 cos2 (n) − 3(cos2 (n) + sin2 (n))


 

= cos(n) [4 cos2 (n) − 3].



2. Ainsi, pour tout entier naturel n, on a tn = n + cos(3n) + 1.
√ √ √
Or, −1 6 cos(3n) 6 1 ⇔ n − 1 6 n + cos(3n) 6 n + 1
√ √ √
⇔ n 6 n + cos(3n) + 1 6 n + 2
√ √ √
⇔ n 6 tn 6 n + 2. Et on a lim n = +∞ donc, par comparaison, lim tn =
n→+∞ n→+∞
+∞.

Corrigé exercice 85 :
1. La suite (kn ) semble converger vers 0.

2. Pour tout entier naturel n, on a −1 6 sin(n) 6 1 ⇔ n2 − 1 6 n2 + sin(n) 6


n2 + 1. De plus, la fonction f : x 7→ x3 − 2x + 5 est définie dérivable surr
R+ , de
2
dérivée x 7→ 3x2 − 2. Cette fonction admet donc un minimum global en sur
r ! 3
+ 2
R et f > 0 d’où, pour tout entier naturel n, n3 − 2n + 5 > 0 et donc
3
n2 − 1 n2 + 1
6 kn 6 .
n3 − 2n + 5 n3 − 2n + 5
3. a. On a lim n2 − 1 = +∞ et lim n3 − 2n + 5 = +∞.
n→+∞ n→+∞
On ne peut donc pas conclure directement.
Or, pour tout entier naturel n :
 
2 1 1
n 1− 2 1− 2
n2 − 1 n n
=  =  .
n3 − 2n + 5 2 5 2 5
n3 1 − 2 + 3 n 1− 2 + 3
n n n n
1 2 5
De plus, lim 1 − 2 = 1. Et lim 1 − 2 + 3 = 1 ainsi que lim n = +∞
n→+∞ n  n→+∞ n n n→+∞
2 5
d’où, par produit, lim n 1 − 2 + 3 = +∞.
n→+∞ n n
n2 − 1
En conclusion, par quotient, lim 3 = 0.
n→+∞ n − 2n + 5
n2 + 1
On montre de même que lim 3 = 0.
n→+∞ n − 2n + 5

b. D’après le théorème des gendarmes, on a lim kn = 0.


n→+∞

Corrigé exercice 86 :
√ √ √
1. Pour tout entier naturel n, on a −1 6 cos(n) 6 1 ⇔ n−1 6 n+cos(n) 6 n+1.
√ √
2 n−1 n+1
Or, n + n + 1 > 0 d’où 2 6 an 6 2 .
n +n+1 n +n+1

Document sous licence libre Creative Commons


53

Livre du professeur - Mathématiques Tle Spécialité - Chapitre 4 : Suites


2. a. On a lim n − 1 = +∞ et lim n2 + n + 1 = +∞.
n→+∞ n→+∞
On ne peut pas conclure directement.
Or, pour tout entier naturel n :

 
1 1
√ n 1− √ 1− √
n−1 n n
= = .

2
 
n +n+1 2
1 1 1 1
n 1+ + 2 n n 1+ + 2
n n n n
1 1 1 √
De plus, lim 1 − √ = 1. Et lim 1 + + 2 = 1, lim n = +∞,
n→+∞ n n→+∞ n n n→+∞


1 1
lim n = +∞ d’où, par produit, lim n n 1 + + 2 = +∞.
n→+∞ n→+∞ n n

n−1
En conclusion, par quotient, lim 2 = 0.
n→+∞ n + n + 1

n+1
On montre de même que lim 2 = 0.
n→+∞ n + n + 1

b. D’après le théorème des gendarmes, on a lim an = 0.


n→+∞

Corrigé exercice 87 :
1. Pour tout entier n > 4, on a −1 6 (−1)n 6 1 et −1 6 cos(n) 6 1 d’où −1 6
(−1)n cos(n) 6 1. D’où 2n − 1 6 2n + (−1)n cos(n) 6 2n + 1.
2n − 1 2n + 1
Or, comme 3 − n < 0, on a alors > cn > .
3−n 3−n
2. a. On a lim 2n − 1 = +∞ et lim 3 − n = −∞.
n→+∞ n→+∞
On ne peut pas conclure directement.
 
1 1
n 2− 2−
2n − 1 n n.
Or, pour tout entier n > 4, =  =
3−n 3 3
n −1 −1
n n
1 3
De plus, lim 2 − = 2 et lim − 1 = −1.
n→+∞ n n→+∞ n
2n − 1
Ainsi, par quotient, lim = −2.
n→+∞ 3 − n
2n + 1
On montre de même que lim = −2
n→+∞ 3 − n

b. D’après le théorème des gendarmes, on a lim bn = −2.


n→+∞

Corrigé exercice 88 :
sin2 (n)
1. Pour tout entier naturel n, on a 1−cos2 (n) = sin2 (n). D’où cn = .
3n2 − 2n + 2 + (−1)n
On a −1 6 (−1)n 6 1 et comme 3n2 − 2n + 2 > 0 (car la fonction x 7→ 3x2 − 2x + 2
est un trinôme du second degré de discriminant négatif), alors 3n2 − 2n + 1 6
3n2 − 2n + 2 + (−1)n 6 3n2 − 2n + 3.

Document sous licence libre Creative Commons


54

Livre du professeur - Mathématiques Tle Spécialité - Chapitre 4 : Suites

1 1 1
D’où > 2 > 2 .
3n2 − 2n + 1 3n − 2n + 2 + (−1) n 3n − 2n + 3
Comme −1 6 sin(n) 6 1 alors 0 6 sin2 (n) 6 1.
sin2 (n) sin2 (n) sin2 (n)
Donc 6 6 .
3n2 − 2n + 3 3n2 − 2n + 2 + (−1)n 3n2 − 2n + 1
sin2 (n) 1
Et enfin 0 6 2 n
6 2 .
3n − 2n + 2 + (−1) 3n − 2n + 1
1
2. On a lim 3n2 − 2n + 1 = +∞ d’où, par quotient, lim = 0.
n→+∞ n→+∞ 3n2 − 2n + 1
D’après le théorème des gendarmes, on a lim cn = 0.
n→+∞

Corrigé exercice 89 :
1. Pour tout entier naturel n, on a 0 6 vn 6 1. Or, comme un > 0, on a alors
0 6 un × vn 6 un .
De même, on montre que 0 6 un × vn 6 vn .

2. Pour tout entier naturel n, on a 0 6 un × vn 6 un et 0 6 un 6 1.


D’où un × vn 6 un 6 1. Or, lim un × vn = 1 donc d’après le théorème des
n→+∞
gendarmes, on a lim un = 1. On montre de même que lim vn = 1.
n→+∞ n→+∞

Corrigé exercice 90 :
π 
1. a. Pour tout entier n > 1, on a 1 − sin2 (n) = cos2 (n) et sin + n = cos(n).
2
1 − sin2 (n) cos2 (n) cos(n)
Ainsi,dn = √ π =√ = √ .
n sin +n n cos(n) n
2
1 1
b. On a −1 6 cos(n) 6 1 d’où − √ 6 dn 6 √ .
n n
1 1
2. On a lim √ = 0 d’où lim − √ = 0.
n→+∞ n n→+∞ n
D’après le théorème des gendarmes, on a lim dn = 0.
n→+∞

Document sous licence libre Creative Commons


55

Livre du professeur - Mathématiques Tle Spécialité - Chapitre 4 : Suites

11 Exercices de synthèse
Corrigé exercice 91 :
1. a. Pour tout entier n > 1, on a :
(n + 1)2 2 1
un+1 n+1 1

n + 1
2
n 2
+ 2n + 1 1+ + 2
vn = = 2 2 = = = n n
un n 2 n 2n2 2
2 n
2 1 2 1
Or, lim = 0 et lim 2 = 0 d’où, par somme, lim 1 + + 2 = 1.
n→+∞ n n→+∞ n n→+∞ n n
2 1
1+ + 2
Ainsi, par quotient, lim n n = 1.
n→+∞ 2 2
 2
1 1
b. Pour tout entier n > 1, on a montré que vn = 1+ . Pour tout entier
2 n
 2
1 n+1 1 1
n > 1, on a 1 + > 1 ⇔ >1 ⇔ 1+ > 1 ⇔ vn > .
n n n 2
 2 r
3 1 1 3 1 3 1
c. On a vn 6 ⇔ 1+ 6 ⇔ < − 1 car 1 + > 0 ⇔ n >
4 2 n 4 n 2 n
1
r .
3
−1
2
1
Or, r ≈ 4, 45 donc le plus petit entier n0 tel que, pour tout n > n0 ,
3
−1
2
3
vn < est n0 = 5.
4
3
d. Pour tout entier n > 5, on a vn 6 .
4
un+1
Or, pour tout entier n > 1, vn = d’où, pour tout entier n > 5, on a
un
un+1 3
6 . Or, comme pour tout entier n > 1, un > 0 alors pour tout entier
un 4
3
n > 5, on a un+1 6 un .
4
 n−5
3
2. a. Soit n > 5. On note Pn la proposition : « un 6 u5 ». On souhaite
4
démontrer que Pn est vraie pour tout n > 5.
Initialisation
 5−5 : Pour n= 5. 5−5
3 3
= 1 donc u5 = u5 . On en déduit que P5 est vraie.
4 4
Hérédité : On considère un entier naturel k > 5 quelconque tel que Pk est
 k−5
3
vraie (hypothèse de récurrence), autrement dit tel que uk 6 u5 . On
4
 k−4
3
souhaite démontrer que Pk+1 est vraie, autrement dit que uk+1 6 u5 .
4
Document sous licence libre Creative Commons
56

Livre du professeur - Mathématiques Tle Spécialité - Chapitre 4 : Suites

3
D’après la question 1.d., uk+1 6 uk . Par hypothèse de récurrence, on a uk 6
 k−5 4 k−4
3 3 3
u5 . D’où uk+1 6 uk 6 u5 .
4 4 4
Ainsi, P5 est vraie et, pour tout entier k > 5, lorsque Pk est vraie, alors Pk+1 est
vraie aussi. Par le principe
 de récurrence, on en déduit que, pour tout n > 5,
n−5
3
Pn est vraie donc un 6 u5 .
4
 n−5
3
b. Pour tout entier n > 5, on a un 6 u5 .
4
 5−5  6−5  n−5
3 3 3
D’où u5 + u6 + . . . + un 6 u5 + u5 + . . . + u5
4 4 4
"  1  n−5 #
3 3
Soit Sn 6 u5 1 + + ... + .
4 4
 n−4
3
 n−5 1 −
3 3 4
c. Pour tout entier n > 5, on a 1 + + . . . + =
4 4 3
1−
" 4
 n−4 #
3
= 4 1− (somme des termes d’une suite géométrique de premier
4
3
terme 1 et de raison ).
" 4
 n−4 #  n−4
3 3
Ainsi, Sn 6 4u5 1 − . Or, 1 − 6 1 d’où Sn 6 4u5 .
4 4

n+1
P n
P
3. Pour tout entier n > 5, on a Sn+1 − Sn = uk − uk = un+1 . Or, comme pour
k=5 k=5
tout entier n > 1, un > 0 alors la suite (Sn ) est croissante. Comme la suite (Sn ) est
croissante et majorée par 4u5 alors elle converge.

Corrigé exercice 92 :
un + 3vn 2un + vn
1. a. Pour tout entier naturel n, on a vn+1 − un+1 = −
4 3
3un + 9vn − 8un − 4vn 5
= = (vn − un ).
12 12
5 5
b. Pour tout entier naturel n, on a wn+1 = vn+1 − un+1 = (vn − un ) = wn .
12 12
5
Donc la suite (wn ) est une suite géométrique de raison q = et de premier
12
terme w0 = v0 − u0 = 10 − 2 = 8.
 n
n 5
Ainsi, pour tout entier naturel n, on a wn = w0 × q = 8 .
12
2un + vn 2un + vn − 3un
2. a. Pour tout entier naturel n, un+1 − un = − un =
3 3

Document sous licence libre Creative Commons


57

Livre du professeur - Mathématiques Tle Spécialité - Chapitre 4 : Suites

−un + vn wn
= = .
3 3 n 
5 n
Or, comme pour tout entier naturel n, wn = w0 × q = 8 , on en déduit
12
que wn > 0. Ainsi, un+1 − un > 0 et donc la suite (un ) est croissante.
un + 3vn un + 3vn − 4vn
Pour tout entier naturel n, on a vn+1 − vn = − vn =
4 4
un − vn −wn
= =
4 4
Or, comme pour tout entier naturel n, wn > 0, alors vn+1 − vn 6 0.
Donc la suite (vn ) est décroissante.
b. Comme la suite (vn ) est décroissante, pour tout entier naturel n, on a vn 6 v0
c’est-à-dire vn 6 10. Or, pour tout entier naturel n, on a wn > 0 c’est-à-dire
vn − un > 0 d’où vn > un . On a donc un 6 vn 6 10 c’est-à-dire un 6 10.
Comme la suite (un ) est croissante, pour tout entier naturel n, on a un > u0
c’est-à-dire un > 2. Or, pour tout entier naturel n, on a vn > un d’où vn >
un > 2 c’est-à-dire vn > 2.
c. La suite (un ) est croissante et majorée par 10 donc elle converge vers un réel `.
La suite (vn ) est décroissante et minorée par 2 donc elle converge vers un réel
`0 .

3. Pour tout entier naturel n, on a wn = vn − un . Ainsi, lim wn = `0 − `.


n→+∞
 n
5 5
Or, comme < 1 alors lim = 0 et donc, par produit, lim wn = 0.
12 n→+∞ 12 n→+∞

On en déduit alors que `0 − ` = 0 c’est-à-dire ` = `0 .

4. a. Pour tout entier naturel n, on a tn+1 = 3un+1 + 4vn+1


2un + vn un + 3vn
=3× +4× = 3un + 4vn = tn .
3 4
Donc la suite (tn ) est constante égale à 3u0 + 4v0 = 6 + 40 = 46.
b. On a lim tn = 46 et lim tn = 3` + 4` = 7` par opérations sur les limites.
n→+∞ n→+∞
46
D’où 7` = 46 ⇔ ` = .
7

Corrigé exercice 93 :
1
1. a. (Sn ) est la suite géométrique de premier terme 1 et de raison . Ainsi, pour
 n 3
1
1−   n 
3 3 1
tout entier naturel n, on a Sn = = 1− .
1 2 3
1−
3
 n  n
1 1 1
b. Comme < 1 alors lim = 0. Ainsi, par somme, on a lim 1− =
3 n→+∞ 3   n  n→+∞ 3
3 1 3
1. Et donc, par produit, il vient lim 1− = .
n→+∞ 2 3 2

Document sous licence libre Creative Commons


58

Livre du professeur - Mathématiques Tle Spécialité - Chapitre 4 : Suites

n+1
P k Pn k n+1
2. a. Pour tout entier naturel n, on a Tn+1 − Tn = k
− k
= n+1 .
k=0 3 k=0 3 3
n+1
Comme > 0, on en déduit que la suite (Tn ) est croissante.
3n+1
b. Pour tout entier naturel n > 1, on a
n+1
P k Pn k
3Tn+1 − Tn = 3 k
− k
k=0 3 k=0 3
   
1 2 n n+1 1 2 n
=3 + + . . . + n + n+1 − + + ... + n
3 32 3 3 3 32 3
 
2 n n+1 1 2 n
= 1 + + . . . + n−1 + n − + 2 + ... + n
3 3 3 3 3 3
   
2 1 n+1 n
=1+ − + ... + n
− n
3 3 3 3
1 1
= 1 + + . . . + n = Sn .
3 3
Sn + Tn
D’où Tn+1 = .
3
c. Soit n ∈ N∗ . On note Pn la proposition : « Tn 6 1 ». On souhaite démontrer
que Pn est vraie pour tout n ∈ N∗ .
Initialisation : Pour n = 1.
0 1 1
T1 = 0 + 1 = 6 1. On en déduit que P1 est vraie.
3 3 3
Hérédité : On considère un entier naturel k > 1 quelconque tel que Pk est
vraie (hypothèse de récurrence), autrement dit tel que Tk 6 1. On souhaite
démontrer que Pk+1 est vraie, autrement dit que Tk+1 6 1.
Sk + Tk 1
On a Tk+1 = . Or, la suite (Sn ) est croissante, car Sn+1 −Sn = n+1 > 0,
3 3
3 3
et converge vers donc, pour tout entier naturel n, Sn 6 . Ainsi, Tk+1 6
2 2
3
+ Tk 5
2 . De plus, par hypothèse de récurrence, on a Tk 6 1 d’où Tk+1 6 6 1.
3 6
Ainsi, P1 est vraie et, pour tout entier k, lorsque Pk est vraie, alors Pk+1 est
vraie aussi. Par le principe de récurrence, on en déduit que, pour tout n ∈ N∗ ,
Pn est vraie donc Tn 6 1.
d. La suite (Tn ) est croissante et majorée par 1 donc elle converge vers un réel `.
3
`+
e. On a ` = 2 ⇔ 3` = ` + 3 ⇔ 2` = 3 ⇔ ` = 3 .
3 2 2 4

Corrigé exercice 94 :
1. On peut construire l’arbre de probabilité ci-dessous.

2. a. Soit n ∈ N∗ . On note Pn la proposition : « pn > 0, 8 ». On souhaite démontrer


que Pn est vraie pour tout n ∈ N∗ .
Initialisation : Pour n = 1.
p1 = 1 > 0, 8. On en déduit que P1 est vraie.

Document sous licence libre Creative Commons


59

Livre du professeur - Mathématiques Tle Spécialité - Chapitre 4 : Suites

Hérédité : On considère un entier naturel k > 1 quelconque tel que Pk est


vraie (hypothèse de récurrence), autrement dit tel que pk > 0, 8. On souhaite
démontrer que Pk+1 est vraie, autrement dit que pk+1 > 0, 8.
Par hypothèse de récurrence, on a pk > 0, 8.
D’où 0, 5pk > 0, 4 ⇔ 0, 5pk + 0, 4 > 0, 8 ⇔ pk+1 > 0, 8.
Ainsi, P1 est vraie et, pour un tout entier k, lorsque Pk est vraie, alors Pk+1 est
vraie aussi. Par le principe de récurrence, on en déduit que, pour tout n ∈ N∗ ,
Pn est vraie donc pn > 0, 8.
b. Pour tout entier n > 1, on a pn+1 − pn = 0, 5pn + 0, 4 − pn = −0, 5pn + 0, 4.
Or, pn > 0, 8 ⇔ −0, 5pn < −0, 4 ⇔ −0, 5pn + 0, 4 < 0.
Donc pn+1 − pn < 0 et alors la suite (pn ) est décroissante.
c. La suite (pn ) est décroissante et minorée par 0,8 donc elle converge vers un réel
`.

3. a. Pour tout entier n > 1, on a vn+1 = pn+1 − 0, 8 = 0, 5pn + 0, 4 − 0, 8


= 0, 5pn − 0, 4 = 0, 5(pn − 0, 8) = 0, 5vn .
Donc la suite (vn ) est géométrique de raison q = 0, 5 et de premier terme
v1 = p1 − 0, 8 = 0, 2.
b. On a alors vn = v1 × q n−1 = 0, 2 × 0, 5n−1 .
D’où pn = 0, 8 + vn = 0, 8 + 0, 2 × 0, 5n−1 .
c. Comme −1 < 0, 5 < 1 alors lim 0, 5n−1 = 0. Ainsi, par produit, lim 0, 2 ×
n→+∞ n→+∞
0, 5n−1 = 0. Et donc, par somme, lim pn = 0, 8.
n→+∞

D’après la formule des probabilités totales, on a :


pn+1 = pn × 0, 9 + (1 − pn ) × 0, 4 = 0, 5pn + 0, 4.

Corrigé exercice 95 :
b
1. On a x = ax + b ⇔ x(1 − a) = b ⇔ x = .
1−a
b un − aun − b
2. Pour tout entier naturel n, on a vn = un − α = un − = .
1−a 1−a

Document sous licence libre Creative Commons


60

Livre du professeur - Mathématiques Tle Spécialité - Chapitre 4 : Suites

b a(un − aun − b)
D’où vn+1 = un+1 − α = aun + b − = = avn .
1−a 1−a
u0 − au0 − b
Donc la suite (vn ) est géométrique de raison a et de premier terme v0 = .
1−a
u0 − au0 − b
3. Pour tout entier naturel n, on a vn = v0 × an = × an .
1−a
u0 − au0 − b b
D’où, un = vn + α = an × + .
1−a 1−a
u0 − au0 − b
4. Si |a| < 1, alors lim an = 0. Ainsi, par produit, on a lim an × =0
n→+∞ n→+∞ 1−a
b
et donc, par somme, lim un = .
n→+∞ 1−a
Si a < −1, alors la suite (an ) diverge et donc la suite (un ) aussi.
Si a > 1, alors lim an = +∞ et donc la suite (un ) diverge et a une limite infinie.
n→+∞

Par produit et par somme, lim un = +∞ lorsque u0 > α et lim un = −∞


n→+∞ n→+∞
lorsque u0 6 α.

Corrigé exercice 96 :
1. a. Pour tout entier naturel n, on a un+1 = an+1 − cn+1
 
1 1 1 1 1 1
= an + b n − bn + c n = (an − cn ) = un .
3 4 4 3 3 3
1
Donc (un ) est une suite géométrique de raison q = et de premier terme
3
u0 = a0 − c0 = 1.
 n  n
1 1
b. Ainsi, pour tout entier naturel n on a un = u0 × q n = 1 × = .
3 3
2. a. Le lapin se trouve forcément dans une des trois galleries A, B ou C.
Ainsi, pour tout entier naturel n, on a an + bn + cn = 1.
4 2 1 2 4
Pour tout entier naturel n, on a vn+1 = bn+1 − = an + bn + cn −
7 3 2 3 7
2 1 2 2 2 4 2 1 4
= an + bn + bn − bn + cn − = (an + bn + cn ) − bn −
3 2 3 3 3 7 3  6 7
2 1 4 1 2 1 4 1
= − bn − = − bn − =− bn − = − vn .
3 6 7 6 21 6 7 6
1
b. (vn ) est donc une suite géométrique de raison q 0 = − et de premier terme v0 =
6  n
4 4 0 n 4 1
b0 − = − . Ainsi, pour tout entier naturel n, vn = v0 ×(q ) = − × − .
7 7 7 6
 n
4 4 4 1
3. Pour tout entier naturel n, on a donc bn = + vn = − × − .
7 7 7 6
Comme an = 1 − bn − cn et an = un + cn alors, pour tout entier naturel n,
1 − bn − cn = un + cn ⇔ 2cn = 1 − bn − un

Document sous licence libre Creative Commons


61

Livre du professeur - Mathématiques Tle Spécialité - Chapitre 4 : Suites

 n  n
4 4 1 1
⇔ 2cn = 1 − + × − −
7 7 6 3
 n  n
3 4 1 1
⇔ 2cn = + × − −
7 7 6 3
 n  n
3 2 1 1 1
⇔ cn = + × − − .
14 7 6 2 3
D’où, pour tout entier naturel n, an = 1 − bn − cn
 n  n  n
4 4 1 3 2 1 1 1
=1− + × − − − × − +
7 7 6 14 7 6 2 3
 n  n
3 2 1 1 1
= + × − + .
14 7 6 2 3
 n  n
1 1 1 1
4. Comme −1 < − < 1 et −1 < < 1 alors lim − = 0 et lim = 0.
6 3 n→+∞ 6 n→+∞ 3
4 3
Ainsi, par opérations sur les limites, on en déduit que lim bn = , lim cn =
n→+∞ 7 n→+∞ 14
3
et lim an = . Après un très grand nombre d’étapes, la probabilité que le lapin
n→+∞ 14
3 4
soit dans la galerie A est , la probabilité qu’il soit dans la galerie B est et la
14 7
3
probabilité qu’il soit dans la galerie C est .
14

Corrigé exercice 97 :
1. On a u1 = 0, 9u0 (1 − u0 ) = 0, 189 donc au début de l’année 2001 il y a 189 tortues.
On a u2 = 0, 9u1 (1 − u1 ) ≈ 0, 138 donc au début de l’année 2002 il y a 138 tortues.

2. a. On a 0 6 1 − un 6 1.
Comme un > 0 alors 0 6 un (1 − un ) 6 un .
D’où 0 6 0, 9un (1 − un ) 6 0, 9un c’est-à-dire 0 6 un+1 6 0, 9un .
b. Soit n ∈ N. On note Pn la proposition : « 0 6 un 6 0, 3 × 0, 9n ». On souhaite
démontrer que Pn est vraie pour tout n ∈ N.
Initialisation : Pour n = 0.
u0 = 0, 3 et 0, 3 × 0, 90 = 0, 3 donc 0 6 u0 6 0, 3 × 0, 90 . On en déduit que P0
est vraie.
Hérédité : On considère un entier naturel k quelconque tel que Pk est vraie
(hypothèse de récurrence), autrement dit tel que 0 6 uk 6 0, 3 × 0, 9k . On
souhaite démontrer que Pk+1 est vraie, autrement dit que 0 6 uk+1 6 0, 3 ×
0, 9k+1 .
On a montré que 0 6 uk+1 6 0, 9uk . Or, par hypothèse de récurrence, on a 0 6
uk 6 0, 3×0, 9k , d’où 0 6 0, 9uk 6 0, 3×0, 9k+1 . Donc 0 6 uk+1 6 0, 3×0, 9k+1 .
Ainsi, P0 est vraie et, pour un entier k, lorsque Pk est vraie, alors Pk+1 est
vraie aussi. Par le principe de récurrence, on en déduit que, pour tout n ∈ N,
Pn est vraie donc 0 6 un 6 0, 3 × 0, 9n .

Document sous licence libre Creative Commons


62

Livre du professeur - Mathématiques Tle Spécialité - Chapitre 4 : Suites

c. Comme −1 < 0, 9 < 1 alors lim 0, 9n = 0. Par produit, on a lim 0, 3 ×


n→+∞ n→+∞
0, 9n = 0. Donc, d’après le théorème des gendarmes, on a lim un = 0. On en
n→+∞
déduit alors que cette population de tortues s’éteindra à long terme.

3. L’algorithme suivant convient :


u ← 0, 3
n←0
Tant que u > 0, 03 :
u ← 0, 9 × u × (1 − u)
n←n+1
Fin Tant que

Corrigé exercice 98 :
1. La figure complétée est la suivante.

p √ √
2. On a d0 = M0 S0 = 5 et d1 = M1 S1 = (5 − 1)2 + (1 − 0)2 = 16 + 1 = 17.

3. Notons (xM2 ; yM2 ) les coordonnées du point M2 .


−−−−→ −−−→
On sait que les points M1 , M2 et S1 sont alignés. Donc les vecteurs M1 M2 et M1 S1
−−−−→ 1 −−−→
sont colinéaires. On sait aussi que M1 M2 = 1 d’où M1 M2 = M1 S1 .
d1
 4 
  √  
−−−→ 4 −−−−→  17  −−−−→ xM2 − 1
Or, M1 S1 d’où M1 M2  1 . On a aussi M1 M2 .
1 √ yM2
17
4 1
D’où xM2 = 1 + √ et yM2 = √ .
17 17
 
4 1
Donc les coordonnées du point M2 sont 1 + √ ; √ .
17 17

Document sous licence libre Creative Commons


63

Livre du professeur - Mathématiques Tle Spécialité - Chapitre 4 : Suites

4. a. La formule saisie dans la cellule B4 est = B3 + (5-B3)/F3. La formule saisie


dans la cellule C4 est = C3 + (A3-C3)/F3.
La formule saisie dans la cellule F4 est = SQRT((D4-B4)ˆ2+(E4-C4)ˆ2).
b. Comme dn représente une longueur, on a dn > 0.
La suite (dn ) est décroissante et minorée par 0 donc elle converge vers un réel
`.
Il semble que ` ≈ 2, 773 166.

Corrigé exercice 99 :
Soit A > 0 un réel. Notons N le plus petit entier supérieur ou égal à A.
N (N − 1)
Le nombre de termes inférieurs strictement à N est 1 + 2 + . . . + (N − 1) = .
2
N (N − 1)
Ainsi, en prenant comme valeur de n0 le plus petit entier supérieur ou égal à ,
2
on a kn > A pour tout n > n0 . Donc lim kn = +∞.
n→+∞

Corrigé exercice 100 :


1. a. Pour tout entier naturel n, on a (vn+1 −un+1 )−(vn −un ) = vn+1 −vn −(un+1 −un ).
Or, comme (vn ) est décroissante, on a vn+1 − vn 6 0 et comme (un ) est crois-
sante, on a un+1 − un > 0. Ainsi, (vn+1 − un+1 ) − (vn − un ) 6 0 et donc la suite
(vn − un ) est décroissante.
b. Comme la suite (vn − un ) est décroissante et converge vers 0 alors elle est
minorée par 0. Ainsi, pour tout entier naturel n, on a vn − un > 0 ⇔ vn > un .

2. a. Pour tout entier naturel n, on a un 6 v0 . Donc la suite (un ) est majorée par
v0 .
b. La suite (un ) est croissante et majorée par v0 donc elle converge vers un réel `.

3. a. Par somme, on a lim vn − un = `0 − `.


n→+∞

b. Or, on sait que lim vn − un = 0 d’où `0 − ` = 0 et donc ` = `0 .


n→+∞

4. a. Commençons par montrer que la suite (un ) est croissante.


Pour tout entier naturel n, on a
n+1
P 1 Pn 1 1
un+1 − un = − = .
k=0 k! k=0 k! (n + 1)!
1
Or, comme (n + 1)! = 1 × 2 × . . . × n × (n + 1) > 0 alors > 0.
(n + 1)!
Donc la suite (un ) est croissante.
Montrons à présent que la suite (vn ) est décroissante.
Pour tout entier naturel n, on a :
 
1 1
vn+1 − vn = un+1 + − un +
(n + 1)! × (n + 1) n! × n
1 1
= un+1 − un + −
(n + 1)! × (n + 1) n! × n

Document sous licence libre Creative Commons


64

Livre du professeur - Mathématiques Tle Spécialité - Chapitre 4 : Suites

1 n! × n − (n + 1)! × (n + 1)
= +
(n + 1)! n! × (n + 1)! × n(n + 1)
−1
=
(n + 1)! × n(n + 1)
Or, pour tout entier naturel n, on a (n + 1)! > 0 et n(n + 1) > 0 d’où
−1
< 0 donc la suite (vn ) est croissante.
(n + 1)! × n(n + 1)
Montrons à présent que lim un − vn = 0.
n→+∞
 
1 1
Pour tout entier naturel n, on a un − vn = un − un + =− .
n! × n n! × n
Or, n! = 1 × 2 × . . . × n donc n! > n donc, par comparaison, lim n! = +∞.
n→+∞
Ainsi, par produit, lim n! × n = +∞.
n→+∞
1
D’où, par inverse, lim = 0. Et donc lim un − vn = 0.
n→+∞ n! × n n→+∞

b. Grâce à la calculatrice, en calculant u9 et v9 , on trouve que la limite commune


de ces deux suites vaut environ 1, 718 282 à 10−6 près.

Corrigé exercice 101 :


1. Une équation de la tangente à f au point d’abscisse xn est y = f 0 (xn )(x−xn )+f (xn ).
Et, pour tout entier naturel n, on a
f 0 (xn )(xn+1 − xn ) + f (xn ) = 0 ⇔ xn+1 f 0 (xn ) − xn f 0 (xn ) + f (xn ) = 0
f (xn )
⇔ xn+1 f 0 (xn ) = xn f 0 (xn ) − f (xn ) ⇔ xn+1 = xn − .
f 0 (xn )
2. a. Pour tout réel x > 0, on a f 0 (x) = 2x.
x2n − 2 2x2 − x2n + 2 x2 + 2
D’où, pour tout entier naturel n, xn+1 = xn − = n = n .
2xn 2xn 2xn
b. La fonction g est définie et dérivable sur ]0; +∞[ comme quotient de fonctions
définies, dérivables et ne s’annulant pas sur cet intervalle.
Et, pour tout x ∈ R :
u0 (x) × v(x) − v 0 (x) × u(x) 2x × 2x − 2(x2 + 2) x2 − 2
g 0 (x) = = = .
[v(x)]2 (2x)2 2x2

Or, pour tout réel x > 0, on a 2x2 > 0 et x2 − 2 > 0 ⇔ x2 > 2 ⇔ x > 2.

Document sous licence libre Creative Commons


65

Livre du professeur - Mathématiques Tle Spécialité - Chapitre 4 : Suites

 √  √ 
Ainsi, la fonction g est décroissante sur 0; 2 et croissante sur 2; +∞ .

√ √ ( 2)2 + 2 4
Elle admet donc un minimum en 2 qui vaut g( 2) = √ = √ =
2 2 2 2
2 √
√ = 2.
2

c. Soit n ∈ N. On note Pn la proposition : « 2 6 xn+1 6 xn ». On souhaite
démontrer que Pn est vraie pour tout n ∈ N.
Initialisation : Pour n = 0.
x20 + 2 22 + 2 6 3 √
x0 = 2 et x1 = = = = donc 2 6 x1 6 x0 . On en déduit
2x0 2×2 4 2
que P0 est vraie.
Hérédité : On considère un entier naturel k quelconque
√ tel que Pk est vraie
(hypothèse de récurrence), autrement dit tel que √2 6 xk+1 6 xk . On souhaite
démontrer que Pk+1 est vraie, autrement dit que 2 6 xk+2 6 xk+1 .

Par hypothèse de √  on a 2 6 x√
récurrence, k+1 6 xk . Or, comme la fonction g
est croissante sur 2; +∞ , on a alors g( 2) 6 g(xk+1 ) 6 g(xk ) c’est-à-dire

2 6 xk+2 6 xk+1 .
Ainsi, P0 est vraie et, pour tout entier k, lorsque Pk est vraie, alors Pk+1 est
vraie aussi. Par le √ principe de récurrence, on en déduit que, pour tout n ∈ N,
Pn est vraie donc 2 6 xn+1 6 xn .

d. La suite √(xn ) est donc décroissante et minorée par 2 : elle converge vers un
réel ` > 2.
`2 + 2 √ √
e. On a ` = ⇔ 2`2 = `2 + 2 ⇔ `2 = 2 ⇔ ` = 2 ou ` = − 2. Mais, pour
2` √ √
tout n ∈ N, un > 2 donc ` = 2.

3. a. On a ∆ = (−1)2 − 4 × 1 × (−1) = 5. Comme √ ∆ > 0, l’équation


√ x2 − x − 1 = 0
1− 5 1+ 5
admet deux solutions réelles : x1 = et x2 = = Φ.
2 2
b. Pour tout réel x > 0, on a f 0 (x) = 2x − 1.
x2n − xn − 1 x2n + 1
D’où, pour tout entier naturel n, xn+1 = xn − = .
2xn − 1 2xn − 1
c. La fonction g est définie et dérivable sur [1; +∞[ comme quotient de fonctions
définies, dérivables et ne s’annulant pas sur cet intervalle. Et, pour tout x > 1,
2x2 − 2x − 2 2(x2 − x − 1)
g 0 (x) = = .
(2x − 1)2 (2x − 1)2
Or, pour tout réel x > 1, on a (2x − 1)2 > 0 et x2 − x − 1 > 0 ⇔ x > Φ.
Ainsi, la fonction g est décroissante sur [1; Φ[ et croissante sur ]Φ; +∞[.
Φ2 + 1
Elle admet donc un minimum en Φ qui vaut g(Φ) = = Φ. (Remarque :
2Φ − 1
Φ2 − 1Φ − 1 = 0 d’où 2Φ2 − Φ2 − Φ − 1 = 0 et donc 2Φ2 − Φ = Φ2 + 1 et par
conséquent Φ(2Φ − 1) = Φ2 + 1 d’où la valeur de g(Φ).)
d. Soit n ∈ N. On note Pn la proposition : « Φ 6 xn+1 6 xn ». On souhaite
démontrer que Pn est vraie pour tout n ∈ N.
Initialisation : Pour n = 0.

Document sous licence libre Creative Commons


66

Livre du professeur - Mathématiques Tle Spécialité - Chapitre 4 : Suites

x20 + 1 22 + 1 5
x0 = 2 et x1 = = = donc Φ 6 x1 6 x0 . On en déduit que
2x0 − 1 2×2−1 3
P0 est vraie.
Hérédité : On considère un entier naturel k quelconque tel que Pk est vraie
(hypothèse de récurrence), autrement dit tel que Φ 6 xk+1 6 xk . On souhaite
démontrer que Pk+1 est vraie, autrement dit que Φ 6 xk+2 6 xk+1 .
Par hypothèse de récurrence, on a Φ 6 xk+1 6 xk . Or, comme la fonction
g est croissante sur [Φ; +∞[, on a alors g(Φ) 6 g(xk+1 ) 6 g(xk ) c’est-à-dire
Φ 6 xk+2 6 xk+1 .
Ainsi, P0 est vraie et, pour tout entier naturel k, lorsque Pk est vraie, alors
Pk+1 est vraie aussi. Par le principe de récurrence, on en déduit que, pour tout
n ∈ N, Pn est vraie donc Φ 6 xn+1 6 xn .
e. La suite (xn ) est donc décroissante et minorée par Φ : elle converge vers un
réel ` > Φ.
`2 + 1
f. On a ` = ⇔ `(2` − 1) = `2 + 1 ⇔ `2 − ` − 1 = 0 ⇔ ` = Φ car, pour tout
2` − 1
n ∈ N, xn > Φ.

Corrigé exercice 102 :


1. Soit n ∈ N. On note Pn la proposition : « un > 0 ». On souhaite démontrer que Pn
est vraie pour tout n ∈ N.
Initialisation : Pour n = 0.
u0 ∈ R∗+ donc u0 > t0. On en déduit que P0 est vraie.
Hérédité : On considère un entier naturel k quelconque tel que Pk est vraie (hypo-
thèse de récurrence), autrement dit tel que uk > 0. On souhaite démontrer que Pk+1
est vraie, autrement dit que uk+1 > 0. On a u2k + a > 0 car un carré est toujours
u2 + a
positif et a est un nombre réel strictement positif. Donc k > 0 par hypothèse
uk
de récurrence, c’est-à-dire uk+1 > 0.
Ainsi, P0 est vraie et, pour tout entier naturel k, lorsque Pk est vraie, alors Pk+1 est
vraie aussi. Par le principe de récurrence, on en déduit que, pour tout n ∈ N, Pn est
vraie donc un > 0.
√ √
(un − a)2 u2n − 2un a + a
2. a. Pour tout entier naturel n, on a =
2u n 2un
√ √
 
1 a
= un + − a = un+1 − a.
2 un

b. D’après √la 2question précédente, pour tout entier naturel n, on a un+1 − a =
(un − a) √
. Or, (un − a)2 > 0 car un carré est toujours positif dans R et
2un √
(un − a)2 √
un > 0 d’après la question 1. Ainsi, > 0 et donc un+1 − a > 0.
2un
  √ √
1 a ( a − un )( a + un )
c. Pour tout n ∈ N, un+1 − un = un + − un =
2 un 2un
√ √
Or, d’après la question 2.b., on a un − a > 0 d’où a − un 6 0.

Document sous licence libre Creative Commons


67

Livre du professeur - Mathématiques Tle Spécialité - Chapitre 4 : Suites

√ √ √
De plus, comme un > 0 alors√ a + un > √ 0. Ainsi, on a ( a − un )( a + un ) 6 0.
( a − un )( a + un )
Et comme, 2un > 0, alors 6 0.
2un
Donc la suite (un ) est décroissante.

3. La suite (un )√est décroissante et minorée par 0 (d’après la question 1., elle est aussi
minorée par a d’après la question 2.b.) donc elle converge vers un réel `.
1 a `2 + a √ √
4. On a ` = `+ ⇔`= ⇔ ` = a ou ` = − a.
2 ` 2`

Or, comme (un ) est minorée par 0, alors ` > 0 et donc ` = a.

Corrigé exercice 103 :


Partie A

1. On a u0 = λr10 + µr20 = λ + µ et u1 = λr11 + µr21 = λr1 + µr2 .


Ainsi, u1 − r1 u0 = λr1 + µr2 − r1 (λ + µ) = µ(r2 − r1 ).
u1 − r1 u0
Donc µ = .
r2 − r1
De même, u1 − r2 u0 = λr1 + µr2 − r2 (λ + µ) = λ(r1 − r2 ).
r2 u0 − u1
Donc λ = .
r2 − r1
r2 u0 − u1 u1 − r1 u0 u0 (r2 − r1 )
2. a. On a λr10 + µr20 = λ + µ = + = = u0 .
r2 − r1 r2 − r1 r2 − r1
Donc P (0) est vraie.
r2 u0 − u1 u1 − r1 u0 u1 (r2 − r1 )
λr11 + µr21 = λr1 + µr2 = r1 + r2 = = u1 .
r2 − r1 r2 − r1 r2 − r1
Donc P (1) est vraie.
b. Comme P (k) est vraie, on a uk = λr1k + µr2k et comme P (k + 1) est vraie, on
a uk+1 = λr1k+1 + µr2k+1 .
uk+2 = auk+1 + buk = a(λr1k+1 + µr2k+1 ) + b(λr1k + µr2k ) = aλr1k+1 + aµr2k+1 +
bλr1k + bµr2k = λr1k (ar1 + b) + µr2k (ar2 + b)
= λr1k × r12 + µr2k × r22 , car r1 et r2 sont solutions de (E).
= λr1k+2 + µr2k+2 .
Ainsi,P (k + 2) est vraie.
c. Ainsi, P (0) et P (1) sont vraies et, pour tout entier naturel k, lorsque P (k) et
P (k+1) sont vraies, alors P (k+2) est vraie aussi. Par le principe de récurrence,
on en déduit que, pour tout n ∈ N, P (n) est vraie donc un = λr1n + µr2n .

3. L’équation caractéristique associée est r2 = r + 1 ⇔ r2 − r − 1 = 0.


On a ∆ = (−1)2 − 4 × 1 × (−1)√= 1 + 4 = 5. Comme
√ ∆ > 0, l’équation r2 = r + 1
1− 5 1+ 5
admet deux solutions r1 = et r2 = .
2 2

Document sous licence libre Creative Commons


68

Livre du professeur - Mathématiques Tle Spécialité - Chapitre 4 : Suites

√ √ √
1+ 5 1+ 5−2 −1 + 5 √
×1−1 −1 + 5
On a donc λ = √2 √ = √ 2 √ = 2
√ = √
1+ 5 1− 5 1+ 5−1+ 5 2 5 2 5

2 2 2 2
√ √ √
1− 5 2−1+ 5 1+ 5 √
1− ×1 1 + 5
et µ = √ 2 √ = √ 2 √ = √2 = √
1+ 5 1− 5 1+ 5−1+ 5 2 5 2 5

2 2 2 2
√ √ !n
−1 + 5 1 − 5
Donc, pour tout entier naturel n, un = λr1n + µr2n = √ +
2 5 2
√ √ !n
1+ 5 1+ 5

2 5 2

Partie B

1. Comme r0 est l’unique solution de l’équation (E), on a r02 = ar0 + b, c’est-à-dire


r02 − ar0 − b = 0. Ainsi, r0 est l’unique racine du polynôme x2 − ax − b = 0 d’où
x2 −ax−b = (x − r0 )2 . Or, (x − r0 )2 = x2 −2r0 x+r02 . D’où x2 −ax−b = x2 −2r0 x+r02 .
Ainsi, par identification, on a −a = −2r0 et −b = r02 . On obtient bien r2 = 2r0 r −r02 .

2. On a u0 = λr00 + µ × 0 × r00 = λ et u1 = λr01 + µ × 1 × r01 = λr0 + µr0 .


u1 − r0 u0
D’où u1 − r0 u0 = λr0 + µr0 − r0 λ = µr0 et alors µ = .
r0
3. Initialisation : On a λr00 + µ × 0 × r00 = λ = u0 Donc P (0) est vraie.
Et λr01 + µ × 1 × r01 = λr0 + µr0 = u1 . Donc P (1) est vraie.
Hérédité : Supposons que P (k) est vraie, c’est-à-dire uk = λr0k + µkr0k , et que
P (k + 1) est vraie, c’est-à-dire uk+1 = λr0k+1 + µ(k + 1)r0k+1 .
On a uk+2 = auk+1 + buk = a(λr0k+1 + µ(k + 1)r0k+1 ) + b(λr0k + µkr0k ), par hypothèse
donc
uk+2 = aλr0k+1 + aµ(k + 1)r0k+1 + bλr0k + bµkr0k
= λr0k (ar0 +b)+µr0k (k(ar0 +b)+ar0 ) = λr0k ×r02 +µr0k (kr02 +r02 −b), car r0 solution de
(E) = λr0k+2 +µr0k (kr02 +r02 +r02 ), car −b = r02 d’après 1. = λr0k+2 +µ(k +2)r0k+2 . Donc
P (k + 2) est vraie. Ainsi, P (0) et P (1) sont vraies et, pour tout entier naturel k,
lorsque P (k) et P (k +1) sont vraies, alors P (k +2) est vraie aussi. Par le principe de
récurrence, on en déduit que, pour tout n ∈ N, P (n) est vraie donc un = λr0n +µnr0n .

4. L’équation caractéristique associée est r2 = 4r − 4 ⇔ r2 − 4r + 4 = 0 ⇔ (r − 2)2 =


0 ⇔ r − 2 = 0 ⇔ r = 2.
Ainsi, l’équation caractéristique admet une unique solution r0 = 2.
3−2×4 −5
On a donc λ = u0 = 4 et µ = = .
2 2
5
Donc, pour tout entier naturel n, un = λr0n + µnr0n = 4 × 2n − n × 2n .
2

Document sous licence libre Creative Commons


69

Livre du professeur - Mathématiques Tle Spécialité - Chapitre 4 : Suites

12 Exercices Préparer le bac


Corrigé exercice 104 :
1. D’après le contexte, on peut supposer que la suite (Tn ) est décroissante. En effet, le
café devrait naturellement se refroidir.

2. Pour tout entier naturel n, on a Tn+1 −Tn = −0, 2(Tn −10) ⇔ Tn+1 = Tn −0, 2Tn +2 =
0, 8Tn + 2.

3. a. Pour tout entier naturel n, un+1 = Tn+1 − 10 = 0, 8Tn + 2 − 10 = 0, 8Tn − 8


= 0, 8(Tn − 10) = 0, 8un .
Ainsi, (un ) est une suite géométrique de raison q = 0, 8 et de premier terme
u0 = T0 − 10 = 70.
b. Pour tout entier naturel n, on a alors un = u0 × q n = 70 × 0, 8n .
D’où Tn = un + 10 = 70 × 0, 8n + 10.
c. Comme −1 < 0, 8 < 1 alors lim 0, 8n = 0.
n→+∞
D’où, par produit, lim 70 × 0, 8n = 0.
n→+∞
Et alors, par somme, lim Tn = lim 70 × 0, 8n + 10 = 10.
n→+∞ n→+∞

4. a. À la fin de l’exécution de cet algorithme, la variable n contient la valeur 4.


b. Au bout de 4 minutes, la température du café sera inférieure ou égale à 40◦ C.

Corrigé exercice 105 :


Partie A
La fonction f est une fonction définie et dérivable sur [0; ∞[ comme produit de fonctions
définies et dérivables sur cet intervalle. Et, pour tout x ∈ [0; ∞[, f 0 (x) = 1×e−x +(−e−x )×
x = e−x (1 − x). Or, pour tout réel x > 0, on a e−x > 0 donc f 0 (x) est du même signe que
1 − x.
Or, 1 − x > 0 ⇔ x 6 1. Donc f 0 (x) > 0 sur [0; 1] et f 0 (x) 6 0 sur [1; +∞[.

Partie B

1. On obtient le graphique suivant.

Document sous licence libre Creative Commons


70

Livre du professeur - Mathématiques Tle Spécialité - Chapitre 4 : Suites

2. Soit n ∈ N. On note Pn la proposition : « 1 > un > 0 ». On souhaite démontrer que


Pn est vraie pour tout n ∈ N.
Initialisation : Pour n = 0.
u0 = 1 donc on a bien 1 > u0 > 0. On en déduit que P0 est vraie.
Hérédité : On considère un entier naturel k quelconque tel que Pk est vraie (hy-
pothèse de récurrence), autrement dit tel que 1 > uk > 0. On souhaite démontrer
que Pk+1 est vraie, autrement dit que 1 > uk+1 > 0.
Par hypothèse de récurrence, on a 1 > uk > 0. Or, comme la fonction f est croissante
sur [0; 1], on en déduit que f (1) > f (uk ) > f (0) c’est-à-dire e−1 > uk+1 > 0.
Donc 1 > uk+1 > 0.
Ainsi, P0 est vraie et, pour tout entier naturel k, lorsque Pk est vraie, alors Pk+1 est
vraie aussi. Par le principe de récurrence, on en déduit que, pour tout n ∈ N, Pn est
vraie donc 1 > un > 0.

3. Montrer que (un ) est décroissante revient à montrer que pour tout entier naturel
n, un+1 6 un , c’est-à-dire f (un ) 6 un . Comme un > 0, nous allons prouver que
f (x) 6 x pour tout réel x > 0. On a −x < 0 d’où e−x < e0 par croissance de la
fonction exponentielle sur ] − ∞; 0]. Ainsi, on a e−x < 1 et comme x > 0, alors
xe−x < x. La suite (un ) est donc bien décroissante.

4. a. La suite (un ) est décroissante et minorée par 0, donc elle converge.


b. On a xe−x = x ⇔ xe−x − x = 0 ⇔ x(e−x − 1) = 0
⇔ x = 0 ou e−x − 1 = 0 ⇔ x = 0 ou −x = 0 ⇔ x = 0.
Ainsi, la limite de la suite (un ) vaut 0.

Partie C
u←1
S←u
Pour k variant de 1 à 100 :
u ← u × e−u
S ←S+u
Fin Pour

Document sous licence libre Creative Commons


71

Livre du professeur - Mathématiques Tle Spécialité - Chapitre 4 : Suites

Corrigé exercice 106 :


Partie A
1 3 1 3 3 5
1. u1 = − u20 + 3u0 − = − × 22 + 3 × 2 − = −2 + 6 − =
2 2 2 2 2 2
 2
1 3 1 5 5 3 25 15 3 23
u2 = − u21 + 3u1 − =− × +3× − =− + − =
2 2 2 2 2 2 8 2 2 8
2. On a u3 ≈ 2, 99219 et u4 ≈ 2, 99997.

3. La suite (un ) semble être croissante et converger vers 3.

Partie B
1 3 1 9
1. Pour tout entier naturel n, vn+1 = un+1 − 3 = − u2n + 3un − − 3 = − u2n + 3un −
2 2 2 2
1 2 1 1 1 9
et − vn = − (un − 3)2 = − (u2n − 6un + 9) = − u2n + 3un − .
2 2 2 2 2
1 2
Donc on a bien vn+1 = − vn .
2
2. Soit n ∈ N. On note Pn la proposition : « −1 6 vn 6 0 ». On souhaite démontrer
que Pn est vraie pour tout n ∈ N.
Initialisation : Pour n = 0.
v0 = u0 − 3 = 2 − 3 = −1 donc −1 6 v0 6 0. On en déduit que P0 est vraie.
Hérédité : On considère un entier naturel k quelconque tel que Pk est vraie (hy-
pothèse de récurrence), autrement dit tel que −1 6 vk 6 0. On souhaite démontrer
que Pk+1 est vraie, autrement dit que −1 6 vk+1 6 0.
Par hypothèse de récurrence, on a −1 6 vk 6 0. D’où 0 6 vk2 6 1 car la fonction
1 1 1
carrée est décroissante sur [−1; 0]. Donc − 6 − vk2 6 0 ⇔ − 6 vk+1 6 0.
2 2 2
Ainsi, P0 est vraie et, pour tout entier naturel k, lorsque Pk est vraie, alors Pk+1 est
vraie aussi. Par le principe de récurrence, on en déduit que, pour tout n ∈ N, Pn est
vraie donc −1 6 vn 6 0.
 
1 2 1
3. a. Pour tout entier naturel n, on a vn+1 − vn = − vn − vn = −vn vn + 1 .
2 2
Document sous licence libre Creative Commons
72

Livre du professeur - Mathématiques Tle Spécialité - Chapitre 4 : Suites

1
b. Pour tout entier naturel n, on a −1 6 vn 6 0 d’où 0 6 −vn 6 1 et 0 < 6
2
vn 6 1. Donc vn+1 − vn > 0 et alors la suite (vn ) est croissante.

4. La suite (vn ) est croissante et majorée par 0, donc elle converge.


 
1 2 1 2 1 1
5. ` = − ` ⇔ ` + ` = 0 ⇔ ` ` + 1 = 0 ⇔ ` = 0 ou ` + 1 = 0 ⇔ ` = 0 ou
2 2 2 2
` = −2.
Or, pour tout entier naturel n, −1 6 vn 6 0, donc ` = 0.

6. Pour tout entier naturel n, on a un = vn + 3. Or, comme (vn ) converge, alors (un )
converge aussi. Par somme, on a lim un = 3. On a montré que la suite (un ) est
n→+∞
croissante et admet pour limite 3. Nos conjectures sont donc bien validées.

Corrigé exercice 107 :


u0 + 2 2+2 4
1. a. u1 = = = . On effectue les mêmes types de calculs pour les
2u0 + 1 4+1 5
autres valeurs et on obtient u1 = 0, 8, u2 ≈ 1, 08, u3 ≈ 0, 98 et u4 ≈ 1, 01.

b. u1 − 1 = −0, 2 est négatif et (−1)1 = −1 est négatif aussi.


u2 − 1 ≈ 0, 08 est positif et (−1)2 = 1 est positif aussi.
u3 − 1 ≈ −0, 02 est négatif et (−1)3 = −1 est négatif aussi.
u4 − 1 ≈ 0, 01 est positif et (−1)2 = 1 est positif aussi.
un + 2 un + 2 − 2un − 1
c. Pour tout entier naturel n, un+1 − 1 = −1 = =
2un + 1 2un + 1
−un + 1
.
2un + 1
d. Soit n ∈ N. On note Pn la proposition : « un − 1 a le même signe que (−1)n ».
On souhaite démontrer que Pn est vraie pour tout n ∈ N.
Initialisation : Pour n = 0.
u0 − 1 = 1 est positif et (−1)0 = 1 est aussi positif. Donc P0 est vraie.
Hérédité : On considère un entier naturel k quelconque tel que Pk est vraie
(hypothèse de récurrence), autrement dit tel que uk − 1 a le même signe que
(−1)k . On souhaite démontrer que Pk+1 est vraie, autrement dit que uk+1 − 1
a le même signe que (−1)k+1 .

Document sous licence libre Creative Commons


73

Livre du professeur - Mathématiques Tle Spécialité - Chapitre 4 : Suites

Par hypothèse de récurrence, uk − 1 a le même signe que (−1)k .


Montrons que uk+1 − 1 est du signe contraire de uk − 1.
−uk + 1 −(uk − 1)
On a uk+1 − 1 = = .
2uk + 1 2uk + 1
Or, −(uk − 1) est du signe contraire de uk − 1 et 2uk + 1 est positif car d’après
l’énoncé, pour tout n ∈ N, un > 0.
−(uk − 1)
Donc est du signe contraire de uk − 1 c’est-à-dire uk+1 − 1 est
2(uk − 1) + 3
du signe contraire de uk − 1. Donc uk+1 − 1 a le même signe que (−1)k+1 .
Ainsi, P0 est vraie et, pour tout entier naturel k, lorsque Pk est vraie, alors
Pk+1 est vraie aussi. Par le principe de récurrence, on en déduit que, pour tout
n ∈ N, Pn est vraie donc un − 1 a le même signe que (−1)n .

2. a. Pour tout entier naturel n :


un + 2 un + 2 − (2un + 1)
−1
un+1 − 1 2un + 1 2un + 1 −un + 1 2un + 1
vn+1 = = = = ×
un+1 + 1 un + 2 un + 2 + 2u n + 1 2un + 1 3un + 3
+1
2un + 1 2un + 1
−un + 1
= .
3un + 3
1 1 un − 1 −un + 1
b. Pour tout entier naturel n, − vn = − × = = vn+1 .
3 3 un + 1 3un + 3
1
Donc (vn ) est une suite géométrique de raison q = − et de premier terme
3
u0 − 1 2−1 1
v0 = = = .
u0 + 1 2+1 3
 n
n 1 1
Ainsi, pour tout entier naturel n, on a vn = v0 × q = × − .
3 3
 n
1 1
1+ × −
1 + vn 3 3
c. Pour tout entier naturel n, on a donc un = =  n .
1 − vn 1 1
1− × −
3 3
 n
1 1
Or, comme −1 < − < 1 alors lim − = 0.
3 n→+∞ 3
 n
1 1 1
D’où, par produit, lim × − = 0. Ainsi, par somme, lim 1 + ×
n→+∞ 3
 n  3 n n→+∞ 3
1 1 1
− = 1 et lim 1 − × − = 1.
3 n→+∞ 3 3
 n
1 1
1+ × −
3 3
Donc, par quotient, lim  n = 1.
n→+∞ 1 1
1− × −
3 3

Document sous licence libre Creative Commons

Vous aimerez peut-être aussi